• Shuffle
    Toggle On
    Toggle Off
  • Alphabetize
    Toggle On
    Toggle Off
  • Front First
    Toggle On
    Toggle Off
  • Both Sides
    Toggle On
    Toggle Off
  • Read
    Toggle On
    Toggle Off
Reading...
Front

Card Range To Study

through

image

Play button

image

Play button

image

Progress

1/249

Click to flip

Use LEFT and RIGHT arrow keys to navigate between flashcards;

Use UP and DOWN arrow keys to flip the card;

H to show hint;

A reads text to speech;

249 Cards in this Set

  • Front
  • Back
A 43-year-old woman with a long history of schizophrenia complains of a loss of night vision. What medication is most likely responsible?
High doses of thioridazine (Mellaril, a low-potency antipsychotic) are associated with irreversible pigmentation of the retina, leading initially to symptoms of night vision difficulty and ultimately to blindness.
A 28-year-old man with a history of a psychiatric admission 6 months previously is seen in the emergency department with a painful erection, which he says has persisted for 18 hours. What is the best next step?
This priapism is most likely caused by trazodone (Desyrel).

One treatment is epinephrine injected into the corpus of the penis
A 57-year-old woman complains of feeling dizzy when she gets up in the morning and when standing. She takes imipramine (Tofranil) each evening for depression. What is the most likely cause of her symptoms?
The mechanism for orthostatic hypotension caused by tricyclic/heterocyclic antidepressants (TCAs) is α-adrenergic blockade.

Remember anti-HAM AEs of the TCAs: (Histaminergic, adrenergic & muscarinic)

Other TCAs: imipramine (tofranil), amitriptyline (Elavil), Trimipramine (surmontil), nortriptyline (Pamelor), Desipramine (norpramin), clomipramine (anafranil), Doxepin (sinequan)

Note: Nortriptyline (Pamelor) is least likely to cause orthostatic hypotension
A 34-year-old man is seen in the emergency department with a headache, dizziness, and blood pressure of 210/150 mm Hg. He has no medical problems, states that he feels fine, and says that last night he even had a nice meal with wine. What medication is he most likely taking?
This patient probably experienced a hypertensive crisis induced by an interaction between the wine and phenelzine (Nardil), a MAOI.

Other MAOIs: Tranylcypromine (Parnate), isocarboxazid (Marplan)

First step in treatment: IV phentolamine if in ED
pts can also carry 10mg tablet of nifedipine with them, its α-blocking properties will act to lower BP when taking it sublingually
A 22-year-old college student with a history of depression is being treated with sertraline (Zoloft). He enjoys drinking beer on the weekends. What side effect is most likely to occur?
Sexual dysfunction is a very common side effect of SSRI medications (25-30%)

Other SSRIs: Fluoxetine (Prozac), Paroxetine (Paxil), Fluvoxamine (Luvox), Citalopram (Celexa), Escitalopram (Lexapro)
Because of the side effects of his original antidepressant, the college student in the previous question is switched to another agent. He comes to the emergency department several days later with muscle spasms, confusion, fever, tachycardia, and hypertension. What's the likely cause?
This patient was likely switched from a SSRI, sertraline (Zoloft), to a MAOI, such as phenelzine (Nardil). Because both agents increase serotonin levels, 5 weeks should elapse between discontinuation of one medication and initiation of the other. The danger is very serious serotonin syndrome, which has features similar to those of NMS.
A 17-year-old adolescent suffers from bulimia nervosa and is very depressed. She is also suffering from insomnia and apathy. Which medication should be avoided?
Seizure disorders and eating disorders are contraindications for bupropion (Wellbutrin) because of its possible lowering of the seizure threshold and its anorectic effects.
A 32-year-old woman has been taking medication (the name of which she does not remember) for her psychiatric condition. She complains of excessive thirst and urinating “all the time.” What is the most likely diagnosis?
This patient has symptoms of diabetes insipidus, a side effect of lithium used in the treatment of bipolar disease.
A 29-year-old man who “hears voices” at times complains of fever and chills. His temperature is 102°F (38.9°C) with no findings of infection. His white blood cell count is 800 cells/mm3. Which medication is most likely responsible?
This individual has neutropenic fever as a result of agranulocytosis, a side effect of the atypical antipsychotic agent clozapine (Clozaril)
A 38-year-old woman is admitted to the hospital for an elective hysterectomy. On hospital day 3, she experiences auditory and visual hallucinations, has tremors, and is agitated. What would be the best therapy?
This woman is probably experiencing either alcohol or benzodiazepine withdrawal; in either case, benzodiazepines would be the treatment.
A 35-year-old African American woman with bipolar disorder delivers a male newborn who has spina bifida. What is the most likely etiology?
This woman was likely taking valproic acid (Depakene/ Depakote), a mood stabilizer used in treating bipolar disorder, which increases the risk for teratogenicity (eg, a neural tube defect).
A 39-year-old man tries to commit suicide by taking an overdose of amitriptyline tablets. He is rushed to the emergency room where resuscitation is attempted but fails. What is most likely to be noted during the attempted resuscitation or the autopsy?
A tricyclic antidepressant (TCA) overdose can lead to increased QT intervals and ultimately to cardiac dysrhythmias.

TCAs: Imipramine (Tofranil), amitripyline (Elavil), Trimipramine (Surmontil) Nortripyline (Pamelor), Desipramine (Norpamin) Clomipramine (Anafranil), Doxepin (Sinequan)
A 25-year-old man with bipolar disorder took too many pills, had two seizures, and is now in a coma. What's the next step?
Dialysis is used to treat lithium toxicity when it is severe and life- threatening, such as causing seizures or coma.
A 38-year-old schizophrenic woman feels restless and cannot sit still; her physician states that this behavior is caused by her medication. What's the next step?
Akathisia (restlessness) can be treated with propranolol.

Akathisia is an extrapyramidal side effect commonly caused by the high potency antipsychotics due to the anti-dopaminergic effects

high potency antipsychotics = Haloperidol (Haldol), Fluphenazine (Prolixin), Trifluoperazine (Stelazine), Perphenazine (Trilafon), Pimozide (orap)
A 32-year-old woman with panic disorder and anxiety took an overdose of diazepam and is taken to the emergency department with somnolence and hypoventilation. What's the next step?
A benzodiazepine overdose can be treated with flumazenil, which is a benzodiazepine antagonist.
A 30-year-old man being treated for schizophrenia complains of tremor and a slow gait. What's the next step?
The parkinsonian-like symptoms of neuroleptic agents are treated with amantadine or levodopa.
definition of anhedonia
Loss of interest or pleasure in activities that were previously pleasurable.
What does SIGECAPS stand for?
S—sleep changes
I—(decreased) interest
G—(excessive) guilt
E—(decreased) energy
C—(decreased) concentration
A—appetite changes
P—psychomotor agitation or retardation
S—suicidal ideation
A 70-year-old woman presents to her primary care provider complaining of fatigue for the past 7 weeks. She admits to difficulty falling asleep, a poor appetite with a 10-lb weight loss, and thoughts of wanting to die. She admits to having had symptoms similar to these on several occasions in the past, but “never this bad.” Her medical problems include asthma and high cholesterol. She uses an albuterol inhaler only as needed. What symptoms is necessary in order to make a diagnosis of major depressive disorder?
Although a change in appetite, decreased energy, fatigue, and suicidal ideation are all criteria used in diagnosing major depressive disorder, one of the symptoms must be either a depressed mood or anhedonia.
A 44-year-old woman comes to your office for a follow-up visit. She recently received a diagnosis of major depressive disorder and began treatment with citalopram (Celexa, an SSRI) 6 weeks ago. She claims to feel “happy again,” without further depression, crying spells, or insomnia. Her appetite has improved, and she has been able to focus at work and enjoy time with her family. Although she experienced occasional headaches and loose stools at the beginning of her treatment, she no longer complains of any side effects. Which of the following is the most appropriate next step in her treatment?
The proper strategy in the management of an episode of major depression that has recently remitted is to continue treatment at the same dose if it can be tolerated. Early discontinuation of medication can lead to an early relapse. A general rule of thumb is, “The dose that got you better will keep you well.” A reasonable duration for continuing the medication is 6 to 9 months.
What side effects common to SSRIs is the woman in the previous question most likely to complain of in the future?
Although activation (causing insomnia), gastrointestinal symptoms (including nausea), and tremor are common side effects of SSRIs, only sexual dysfunction generally occurs later in the treatment course (after weeks to months).
The same woman in the previous questions is seen 1 year later for a return visit. She has remained on the citalopram (Celexa, an SSRI) at the same dose, and she is tolerating it well, but she worries about “always having to take medication.” What is her risk of recurrence if she is not maintained on antidepressant medication?
The recommendations for maintenance therapy in major depres- sive disorder should be made on a case-by-case basis. However, the illness tends to run a chronic course, especially if treatment is discontinued. Indeed, 50% to 85% of individuals will suffer from at least one further episode, most likely within 2 to 3 years.
definition of loose associations
Thoughts that are not connected to one another or illogical answers to questions.
definition of tangentiality
Thoughts can be connected to each other although the patient does not come back to the original point or answer the question.
what are some examples of the negative symptoms of schizophrenia?
Affective flattening, alogia (diminished flow and spontaneity of speech), and avolition (lack of initiative or goals).
What are some examples of the positive symptoms of schizophrenia?
Ideas of reference, grossly disorganized speech or behavior, delusions (such as paranoia), and hallucinations.
A 46-year-old man presents with a long-standing belief that his thoughts are being taken from his head and used to create a blockbuster movie. He is certain that the government is involved because they often communicate with him through a microchip they have implanted in his brain. Although he feels frustrated at being taken advantage of, he denies any significant depressive symptoms and is often able to enjoy playing cards with his peers at the group home. What's the most likely dx?
The most likely diagnosis for this man is schizophrenia. He has been suffering from psychotic symptoms including delusions and auditory hallucinations for more than 6 months. Although he can have brief periods of depressed mood, he does not have a history of major mood disorder.
A 78-year-old man presents with 4 weeks of significant depression following the sudden, unforeseen death of his wife of 35 years. He reports difficulty sleeping, a 10-lb weight loss, frequent crying spells, and profound guilt over surviving her. For the last several days, he has been convinced that his body is literally decaying. He admits to seeing his wife’s face during the day, as well as hearing her voice telling him to kill himself and join her. What's the most likely dx?
The most likely diagnosis for this man is major depression with psychotic features. Significant depression and neurovegetative symptoms are present, as well as delusions and auditory and visual hallucinations. Although he has mood symptoms and psychotic symptoms, his history is consistent with major depression because his mood symptoms preceded his psychotic symptoms.
A 54yo, psychotic, homeless man is brought into the emergency department by the police for medical clearance, after being picked up for aggressive behavior and disorderly conduct. He has a history of alcoholism, substance abuse, as well as poorly controlled diabetes and hypertension. What symptoms are most specific to a diagnosis of schizophrenia, as opposed to other etiologies of psychosis?
the presence of a bizarre delusion is the most specific to schizophrenia. Only one psychotic symptom is needed to diagnose schizophrenia if there are bizarre delusions, auditory hallucinations commenting on the patient, or two or more voices speaking to each other.
A 27-year-old woman states that for approximately 6 months she has believed that Michael Jackson is in love with her. She insists that he has professed his intentions to marry her through messages in his song lyrics. She has written numerous letters to him and loitered around his home, resulting in several arrests. She is irritated because, although he won’t meet with her in person, he often calls her name outside her window when no one else is around. For the past several weeks, she has slept approximately only 2 hours a night but still has enough energy to continuously redecorate her apartment in preparation for her wedding to Mr. Jackson. She admits to feeling “on top of the world” because Michael Jackson has chosen her and that she “can’t stop talking about it.” Most likely dx?
The most likely diagnosis for this woman is schizoaffective disorder. She describes a 6 month history of ideas of reference, delusions, and auditory hallucinations. In addition, she has had clear manic symptoms for the past month, including an elevated mood, a decreased need for sleep, increased energy, increased goal-directed activities, and talkativeness. Although she has symptoms consistent with schizophrenia, she has had a significant episode of mood disorder during her psychotic illness. Her psychotic symptoms, which preceded and occurred in the absence of mood symptoms, make primary mood disorder (mania) with psychotic features less likely.
A 28-year-old man describes a persistent fear of speaking in public. Although he does not have difficulty with one-on-one situations, when giving a lecture he becomes extremely anxious, worrying that he will be humiliated. He relates one episode in which he was forced to speak at the last minute, which resulted in his experiencing panic, shaking, abdominal cramps, and a fear that he would defecate on himself. Because of this problem, he has been held back from promotion at his place of business. most likely diagnosis?
The most likely diagnosis for this man is social phobia. Although he suffers from panic attacks, they are not unprovoked as in panic disorder because they occur in response to public speaking. His fear is not of having further attacks but rather of being embarrassed or humiliated. For generalized anxiety disorder the anxiety and worry is not confined to being embarrassed as in social phobia, rather it involves excessive anxiety and worry about a number of events or activities. Specific phobia is characterized by an excessive fear that is cued by the presence or anticipation of a specific object or situation (eg, flying, animals) rather than social or performance situations.
A 40-year-old woman presents with complaints of not being able to leave her house. For the past 5 years, she has had increasing difficulty traveling far from home. She constantly worries that she will not be able to get help if she “freaks out.” In fact, she has had numerous unprovoked episodes of intense fear, associated with shortness of breath, chest pain, diaphoresis, and dizziness, that lasted for 20 minutes. She is convinced that if she drives too far from home, she will have an attack and not be able to obtain help. Whats the most likely diagnosis?
This woman most likely has panic disorder with agoraphobia. She experiences recurrent spontaneous panic attacks and between attacks worries about having further attacks. She avoids driving away from her home for fear of being unable to obtain help in the event of an attack.

SSRIs or other antidepressants, in combination with cognitive behavioral therapy (CBT), are used in the pharmacologic treatment of panic disorder. If benzodiazepines are also administered, they should be used in as low a dose and for as short a time as possible.
A 25-year-old woman describes a lifelong history of being “scared of heights.” She becomes uncomfortable when higher than three stories and whenever traveling or shopping becomes preoccupied with knowing the heights of buildings. On finding herself at a significant elevation, she has severe anxiety symptoms such as trembling, lightheadedness, numbness and tingling, and a fear of dying. Whats the most likely diagnosis?
Specific phobia is the most likely diagnosis for this woman. Although she has panic attacks, they are not unexpected and result from being in a high place. Her fears are actually of a situation (heights) rather than of having further panic attacks.
The chief complaint of a 33-year-old man is, “I’m going to have a heart attack like my father.” He explains that his father died of a myocardial infarction at 45 years of age. He is convinced that he is experiencing angina attacks consisting of nervousness, sweating, palpitations, flushing, and numbness in his hands and lasting approximately 5 minutes. He is anxious about having these symptoms and, despite negative results from a cardiology workup, remains certain that he will suffer a heart attack. His behavior and lifestyle have not been otherwise affected. What's the most likely diagnosis?
The most likely diagnosis for this man is panic disorder without agoraphobia. He displays characteristic features of panic attacks, such as recurrent episodes of anxiety associated with physical symptoms. These episodes are spontaneous, and he worries about the consequences of having an additional attack, namely, a myocardial infarction.

SSRIs or other antidepressants, in combination with cognitive behavioral therapy (CBT), are used in the pharmacologic treatment of panic disorder. If benzodiazepines are also administered, they
should be used in as low a dose and for as short a time as possible.
18yo M presents with 3 days of an irritable mood, decreased sleeping, talkativeness, increased energy, and distractibility. He has no personal or family psychiatric history and no current medical problems. His mental status examination is remarkable for psychomotor agitation and an irritable affect. He is paranoid but denies delusions or hallucinations. His physical examination is notable for a slightly elevated pulse rate and blood pressure as well as markedly dilated pupils bilaterally. The result of his urine toxicology screening is positive for cocaine. Most likely dx?
The most likely diagnosis for this man is substance (cocaine)- induced mood disorder. Although he presents with classic manic symptoms (irritable mood, decreased sleep, etc), he has no psychiatric or family history of mood disorder. His physical examination reveals several findings not necessarily consistent with mania, namely, elevated vital signs (pulse rate and blood pressure) and dilated pupils. The important factor in this case is his obvious cocaine use, which can produce symptoms mimicking those of acute mania.
39yo F presents with 1 month of a gradually worsening depressed mood, with increased sleeping, low energy, and difficulty concentrating, but no appetite or weight changes. Her medical history is significant for multiple sclerosis, but she is currently not taking any medication. Her mental status examination is notable for psychomotor slowing and a depressed affect. Her physical examination demonstrates several different sensory and motor deficits. most likely dx?
The most likely diagnosis in this case is mood disorder due to a general medical condition, namely, multiple sclerosis. Although this woman displays the characteristic symptoms of an episode of major depression (depressed mood, increased sleeping, low energy), she does not exhibit the appetite or weight changes commonly seen in this illness. Steroids can often cause mood symptoms such as depression or mania, but she is currently not taking any medication. The results of her physical examination are also consistent with a flare-up of her multiple sclerosis and demonstrate a temporal relationship to her depression. The central nervous system white matter lesions seen on imaging are known to be associated with a depressive state.
52yo M executive presents with an onset of depression, early-morning awakening, decreased energy, distractibility, anhedonia, poor appetite, and weight loss for the past 3 months. His symptoms began shortly after he suffered a myocardial infarction. Although he did not experience significant sequelae, he has felt less motivated and fulfilled in his life and work, believing that he is now “vulnerable.” As a result, he does not push himself as he used to, and his output is beginning to decline. Most likely dx?
The most likely diagnosis for this man is major depression. He has symptoms typical of the disorder, both depressed mood and neurovegetative symptoms lasting for more than 2 weeks. Although his condition was preceded by a heart attack, it was not a physiologic cause of his depression. Rather, his medical illness (and subsequent feeling of vulnerability) was the stressor that brought on his episode of depression.
80yo F without a psychiatric history is examined after a left-sided cerebral vascular accident has left her paralyzed on her right side. Since her stroke, she complains of an absence of pleasure in anything that she formerly enjoyed. She describes frequent crying spells, increased sleeping, a decreased appetite with weight loss, and feelings of hopelessness and helplessness. Most likely dx?
The most likely diagnosis in this case is mood disorder caused by a general medical condition, namely, a cerebral vascular accident. The patient has obvious symptoms of a depressive illness, including anhedonia. These symptoms also have a clear temporal relationship to her stroke, which has left her with significant motor deficits. Cerebral vascular events, especially those affecting the left frontal region (where the stroke occurred), are well known to result in depression.
36yo M with a past h/o a major depressive episode is brought into the ED by the police after stopping traffic on the highway proclaiming that he is “the Messiah.” His wife is contacted who states that he has been walking throughout the house all night for the last 4 nights, talking “nonstop,” and starting many home repair projects that remain unfinished. She believes that he is taking sertraline (Zoloft) for his depression and propranolol for high blood pressure. His blood alcohol level is less than 10, and his urine toxicology screen is negative. Most likely dx?
The most likely diagnosis for this man is bipolar disorder, manic. He has classic symptoms and signs of the illness, such as decreased need for sleep, talkativeness, increased activity, risky behavior, and delusions of grandeur. He also has a history of a major depressive episode and has been taking an antidepressant which likely has caused a switch into his current manic episode. Although he is taking a beta-blocker for hypertension, this would be likely to cause a depressive episode rather than a manic episode.
What medication would be the best choice for treating nonpsychotic mania in a 10-year-old boy?
Mood stabilizers are used to treat bipolar disorder. Lithium is the only medication that has received FDA approval for the treatment of bipolar disorder in youth aged more than 12 years, but Divalproex has a longer safety profile in young individuals given its long history of use for seizures.
A 16-year-old girl has been admitted with a 3-week history of sudden irritability, impulsive buying, and disappearing at night with older men. Her need for sleep is decreased; she has flight of ideas and grandiose thoughts about being an advisor to a presidential candidate. Routine admission labs indicate she is pregnant. What's the best option for treatment?
Atypical antipsychotics are good choices for controlling mania in pregnancy given the strong teratogenic effects of most mood stabilizers. SSRIs will only make the mania worse. The patient will need psychotherapy focused on helping her sort out her feelings about the pregnancy.
Parents of a 10-year-old boy note that their son does well at school or with his family until he is not allowed to do something he wants to do. When this occurs, he will get irritable, impulsively aggressive and agitated for several hours. Once he calms, or gets his way, he is pleasant again. Based on DSM-IV-TR and AACAP guideline this patient should be diagnosed with?
The full DSM-IV-RT criteria must be met to diagnose bipolar dis- order according to AACAP practice parameters. The pattern described best fits oppositional defiant disorder, since the patient can be seen to calm down and be pleasant after he gets his way. A child with bipolar disorder might be seen to have these kinds of outbursts, but the mood dysregulation continues, even after the child “gets his way.” Children in the midst of a bipolar disorder never “calm down and be pleasant.”
What personality disorders fall into cluster A?
Characterized by odd or eccentric behavior. Schizoid, schizotypal, and paranoid personality disorders fall into cluster A.

These clusters can be remembered by the words“mad” (cluster A disorders in which patients display odd or eccentric behavior),“bad” (cluster B disorders in which patients exhibit dramatic or emotional behavior), and “sad” (cluster C disorders in which patients show anxious or fearful behavior).
What PD fall into cluster B?
Characterized by dramatic or emotional behavior. Histrionic, narcissistic, antisocial, and borderline personality disorders fall into cluster B.

Theseclusterscanberememberedbythewords“mad”(clusterAdisorders in which patients display odd or eccentric behavior),“bad”(cluster B disor- ders in which patients exhibit dramatic or emotional behavior), and “sad” (cluster C disorders in which patients show anxious or fearful behavior).
What PD fall into cluster C?
Characterized by anxious or fearful behavior. Obsessive-compulsive, avoidant, and dependent personality disorders fall into cluster C.

Theseclusterscanberememberedbythewords“mad”(clusterAdisorders in which patients display odd or eccentric behavior),“bad”(cluster B disor- ders in which patients exhibit dramatic or emotional behavior), and “sad” (cluster C disorders in which patients show anxious or fearful behavior).
A 48-year-old woman presents to a psychotherapist. The patient lives a very secluded life, largely consumed by working nights as a security guard, and taking care of her elderly mother. She complains of feeling lonely, and is aware that she has a great deal of difficulty relating to other people. Why is it unlikely that she has schizoid personality disorder?
The hallmark of schizoid personality disorder is a detachment and disinterest in social relationships. This patient is clearly distressed by her lack of social relationships.
A woman with schizoid personality disorder was involved in a motor vehicle accident in which she was rear-ended by another car. The driver of the other car refused to take responsibility for the accident and hired a lawyer to provide his defense. The woman spends hours each day thinking about the specifics of the accident, including such details as the color of the cars involved and what each party to the accident was wearing. Which of the following defense mechanisms, common to patients with schizoid personality disorder, is the woman using?
Intellectualization is characterized by rehashing events over and over.
80yo M undergoes an evaluation for dementia versus depression. His caregiver describes a history of gradually worsening depressed mood and confusion, with poor appetite, weight loss, poor self-care, and irritability. On his cognitive examination, he is alert and oriented to person and place but not to time. His concentration is impaired, and he displays poor short-term memory despite adequate recall. His effort is poor overall, and he often responds to questions stating, “I don’t know.” What finding is more consistent with a depressive illness than with dementia?
Answer: poor effort during the interview

Difficulty concentrating, decreased self-care, poor appetite, and short-term memory deficits can be seen in both severe depression and dementia in elderly patients. However, during the cognitive examination, patients with depression usually make little effort but have considerable insight into their difficulties, whereas patients with dementia often make considerable effort but display confabulation and little insight into their mistakes.
A 76-year-old woman with a history of major depressive disorder, recurrent, presents to her PCP with a h/o 4 months of increasing depression, a/w terminal insomnia, decreased appetite, 15 lb weight loss, fatigue, difficulty concentrating, and feelings of helplessness. She denies suicidal ideation, hallucinations, or delusions. She feels these symptoms are similar to those she had in the past. Her last episode was 30 years ago, when she was successfully treated with nortriptyline (Pamelor), but she has not taken any psychotropics since that time. She has high BP, DM, and HLD, but she has no other physical complaints, and her PE and labs are unremarkable. After a great deal of discussion, she agrees to restart nortriptyline. When compared with the treatment of her last episode of major depression, which of the following statements is most accurate regarding medication management of this episode?
answer: more drug-drug interactions

Geriatric patients are often taking multiple medications and therefore have a higher likelihood of being affected by drug-drug interactions. They are also more likely to experience significant side effects from medications. Although the drug levels required to
achieve efficacy are similar to those required in younger patients, because of the decreased clearance and metabolism seen in older patients, lower doses are required to reach the same levels. Efficacy of antidepressants remains similar in elderly patients.
The patient in the previous question returns after 2 weeks despite compliance with nortriptyline. Her depressive symptoms have since worsened, with the new onset of suicidal ideation without a plan. Upon further questioning, she admits to feeling that unknown individuals are following her and to hearing noncommand auditory hallucinations call- ing her name and making derogatory comments. Consideration is given to beginning a trial of ECT. Which of the following is the best indication for using ECT in this patient?
answer: psychotic symptoms

The most appropriate treatment for a patient with major depressive disorder with psychotic features (whether elderly or not) remains ECT or an antidepressant/antipsychotic combination. Other indications for ECT are when a rapid response is necessary, such as in an imminently suicidal patient, a catatonic patient, or a patient who is not ingesting adequate food or fluid. Neither recurrent episodes nor the presence of neurovegetative symptoms are an indi- cation for ECT.
You are consulted to evaluate an 84yo widowed woman on the medical ward. She has a prior history of MDD, recurrent, and she was admitted for a syncopal episode. She was found to be extremely malnourished and has not been taking her antidepressant for many months. She describes having all the neurovegetative symptoms of depression and to not eating or drinking for days. When questioned about this, she admits to purposely “starving myself” as she believes that God is punishing her for directly causing the terrorist attacks of September 11, 2001. In fact, she has been “following God’s instructions,” which tell her to kill herself in atonement. What treatment would be the most appropriate for this patient?
answer: ECT therapy only

This woman’s history and presentation are most consistent with major depression, severe, with psychotic features. The treatment of choice for this illness is either ECT or an antidepressant/antipsychotic combination. Given the medical urgency in this case and that medication would take several weeks for significant efficacy, ECT would be preferable. Neither antidepressants nor antipsychotics alone would be as efficacious in an episode of psychotic depression, and psychotherapy with or without medication would not be appropriate in someone with a severe depression as in this case.
What symptom is more likely to be found in a patient with social phobia-related panic attacks than in a patient with a primary panic disorder?
answer: expected occurrence

Panic attacks can occur in both social phobia as well as in patients with panic disorder. In patients with social phobia however these panic attacks occur with specific social situations and can be expected to occur there. In panic disorder however, the attacks may occur at any time.
What is the treatment of choice for social phobia?
A. Behavior therapy
B. SSRIs
C. Electroconvulsive therapy
D. Psychoanalysis
E. Divalproex sodium (Depakote)
behavior therapy

Behavior therapy is the treatment of choice for social phobia. Benzodiazepines can be used to reduce the associated anxiety. A beta-blocker such as propranolol can also help reduce the autonomic hyperarousal that occurs in social settings. In addition, some SSRIs are also useful.
Social phobia differs from specific phobia in what way?
A. Focus or nature of the fear
B. Duration of the illness
C. Absence of panic attacks
D. Degree of avoidance of the situation
E. Recurrent nature of the fear
answer: focus or nature of the fear

Social phobia might be also considered a specific phobia under some circumstances and is very similar in duration, progression, symptoms, and patient avoidance. The real difference is the nature of the fear—in social phobia this is characterized by social situations.
Dread of being embarrassed in public, fear of speaking in public, or fear of eating in public.
social phobia
examples of specific phobias
Dread of a particular object or situation, such as acrophobia (heights), agoraphobia (open places), algophobia (pain), claustrophobia (closed places), xenophobia (strangers), and zoophobia (animals).
Persistent, irrational, exaggerated, and pathologic fear of a specific situation or stimulus that results in conscious avoidance of the dreaded circumstance.
phobia
def of ataxia
A disturbance in gait; the patient cannot remain steady on his feet.
def of dysarthria
A disturbance in speech, which appears slurred, garbled, or unclear.
def of hyperacusis
Hearing that is especially sensitive
def of nystagmus
Rhythmic, oscillating motion of the eyes. This to-and-fro motion is generally involuntary and can occur in the vertical or horizontal plane.
39yo M presents to ED at the behest of his girlfriend, who reports that he has not slept in 3 days. The patient speaks extremely rapidly, and switches topic so frequently as to be incomprehensible. His affect is happy and elevated, but he quickly snaps and becomes belligerent when he is accidentally bumped by a nurse. Which of the following symptoms would most likely distinguish this patient’s presentation from PCP intoxication?
answer: nystagmus

Individuals with mania and PCP intoxication can have hallucinations, display hostility, and have disordered thoughts; nystagmus is commonly associated with PCP use but not with mania
15yo boy is brought to the ED by police due to violent, psychotic behavior. Phencyclidine intoxication is confirmed via urine toxicology. What's the best tx?
IV antihypertensive agents to treat HTN

note (What NOT to do): Low-potency traditional antipsychotics may worsen intoxication syndrome via anticholinergic side effects. Benzodiazepines may delay excretion of the drug. Gastric lavage is contraindicated due to risk of emesis and aspiration, while restraints may lead to muscle breakdown
An obtunded, young woman is discovered by the police sitting in the middle of the street, and is subsequently brought to the ED. She is unable to verbalize any history. What findings would be most indicative of PCP intoxication?
TRIAD of nystagmus, muscle rigidity, and numbness points strongly to PCP intoxication.

Other symptoms that can occur include hypertension or tachycardia, ataxia, dysarthria, seizures or coma, and hyperacusis.

note: The patient can ingest PCP without knowing; it is a substance that is sometimes added to marijuana cigarettes.
A defense mechanism by which an individual deals with emotional conflict or stressors by attributing exaggerated positive qualities to others. For example, a woman being abused and neglected by her husband earnestly states, “He is the best thing that ever happened to me.”
IDEALIZATION
A defense mechanism by which an individual deals with emotional conflict or stressors by substituting behavior, thoughts, or feelings that are diametrically opposed to his or her own unacceptable thoughts or feelings. For example, a woman who is very angry with her husband for having an affair cooks him a nice dinner and acts sweetly toward him.
REACTION FORMATION
The expression of psychological difficulties as physical complaints. Considered a form of regression because being able to verbalize problems instead of making them physical complaints is considered a progressive step. For example, a woman who is upset about the death of her cat develops an intractable headache.
SOMATIZATION
A 37-year-old woman is referred to your office for psychotherapy. She is preoccupied with thoughts of her husband leaving her and being left alone with nobody to take care of her. She feels sad and hopeless when alone, lacks self-confidence, and has great difficulty making decisions. You believe she suffers from a personality disorder. In which cluster does the patient’s most likely diagnosis belong?
Cluster C

Cluster C can be remembered as the “sad” cluster. It includes dependent, avoidant, and obsessive-compulsive personality disorders. The clusters can be remembered by the words “mad” (cluster A—characterized by odd or eccentric behavior), “bad” (cluster B— characterized by dramatic or emotional behavior), and “sad” (cluster C—characterized by anxious or fearful behavior).
You are consulted to evaluate a 45-year-old, married woman who was admitted to the surgical service 2 days ago for an appendectomy. The procedure went well, but she was found to be tearful, stating “I wish I were dead.” On obtaining further history, she is quite cooperative and talkative. She is questioned about her earlier comments, and she states that she “wanted attention, I guess.” She is upset that her husband is not with her in the hospital; she has “never been away from him” for this long since they started dating when the patient was 16 years old. She feels helpless and is having a difficult time being active in her care. She feels overwhelmed regarding her postsurgical and discharge instructions, and the nursing staff has become frustrated with her constant “need for reassurance.” Although at times she is tearful during the interview, she denies prior or recent pervasive depressive or neurovegetative symptoms and is not actively suicidal. What is the most appropriate approach to this patient?
answer: Spend regular, short periods of time with her to discuss discharge planning and aftercare.

This patient displays characteristics consistent with dependent personality disorder. The most effective approach in dealing with a patient with this disorder is to respect her need for attachment and schedule limited but regular appointments with her. Individuals with this illness wish to be taken care of, and therefore will not be proactive in their care; in circumstances such as these, it’s helpful for the physician to be more active. Encouraging the patient to be less dependent in her primary relationship is not only not helpful, but can be damaging as she can feel rejected and become more regressed, upset, and helpless. Encouraging the husband to spend more time at the hospital only continues the cycle of “being taken care of” and doesn’t facilitate the patient’s involvement in postop care.
What would be the most useful psychiatric treatment for the patient in the previous question?
answer: group psychotherapy

Psychotherapies, such as insight-oriented, family, behavioral, and group therapy have been useful in patients with dependent personality disorder. Medications for depression, anxiety, and/or psychosis, or electroconvulsive therapy (ECT), would only be indicated if the patient had a comorbid psychiatric illness, not evident in this patient
A diagnosis of Generalized Anxiety Disorder (GAD) is made for a 41yo F accountant who has had moderate but distressing symptoms for several years. She has strong negative feelings about taking any psychotropic medication, even after being educated about pharmacologic treatment options. What would be the most efficacious treatment option?
Cognitive-behavioral therapy (CBT) with an SSRI, venlafaxine, or buspirone

Because the patient’s symptoms are moderate and she feels so negatively about medication, the best option for psychotherapy is CBT, the most studied evidence-based psychotherapy. Other interventions such as insight-oriented psychotherapy and supportive psychotherapy are less likely to be effective. This should be combined with a nonaddictive medication for anxiety.
38yo M w/o PMHx presents to his PCP with the CC of “I’m having ulcers.” His history, however does not appear to be consistent with gastritis, ulcers, or reflux. After further questioning, he describes ongoing headaches for 8 months, along with difficulty sleeping. Although he denies any specific or recent stressors, he admits to “always being a worry-wart,” worrying about many different aspects of his life. He only has been taking ranitidine over the counter as needed. He drinks one to two glasses of wine 1 to 2 days per week and denies drug use. Based on his likely diagnosis, what additional psychiatric disorders are most likely to also be present in this patient?
dysthymic disorder

This patient appears to suffer from generalized anxiety disorder. There is a high comorbidity in this illness, especially with other anxiety disorders, such as panic disorder and phobias. Dysthymic disorder and major depressive disorder are also very commonly associated conditions. Substance use disorders are also not uncommon in patients with GAD.
45yo F with alcohol dependence in full remission is referred to your practice. She has a history of GAD and was recently prescribed a benzodiazepine by her internist. Which of the following is the most appropriate treatment for her?
switch to SSRI monotherapy

SSRIs are a first-line treatment for GAD. Potentially addictive medications such as benzodiazepines should be avoided in patients with a history of substance dependence. Buspirone is not as effective in patients already exposed to benzodiazepines.
62yo M presents to his PCP at the insistence of his wife. The man states that there is nothing wrong with him, but that he has not been sleeping nearly as much as he used to, often needing less than 2 hours of sleep a night. His wife notes that he has been exceedingly irritable, has been charging excessive amounts of money on their credit cards, and has been talking about running in a marathon, although he has never expressed such an interest before. The patient has no previous psychiatric or medical history. He denies the use of drugs or alcohol. Which of the following courses of action should the physician take first?
perform a complete physical exam

In a patient of this age, with no previous psychiatric or medical history, general medical conditions responsible for this new-onset behavior should be ruled out before an episode of bipolar mania is considered. While admission to the hospital might well be necessary to control the destructive behavior, making the diagnosis first is the primary concern. Both a mood stabilizer and/or an antipsychotic may well also be necessary, but again, if the mood problem is secondary to a general medical condition, treating it may resolve the psychiatric symptoms by itself.
18yo M is brought to the ED by his friends after he started a fight with one of them and subsequently was knocked unconscious. On awakening, the patient states that he was fighting for the “freedom of the world” and that he was told by “the voices in his head” that his friends held the key to winning the battle. The patient is irritable and restless and paces around in the emergency department. He is unable to sit still for the interview and prefers to stand up, keeping his back to the door as he speaks with the physician. The patient’s friends state that he has been withdrawing from them and his schoolwork over the past 18 months and that they think he has become “odd.” They report that his mood has become irritable only over the past several days. They say that he hoards random scraps of paper in his room and that his grades have dropped dramatically over the past 6 months. The results of a toxicology screening are negative, as are those of a physical examination, although the examination was limited because of poor cooperation on the part of the patient. Which of the following is the most likely diagnosis?

A. Bipolar disorder, manic
B. Mood disorder secondary to a general medical condition
C. Schizoaffective disorder
D. Schizophrenia
E. Substance-induced mood disorder
Schizophrenia

This patient is currently experiencing a frank episode of psychosis, which appears to most recently have escalated with the appearance of paranoia (standing with his back to the door as he speaks with the physician) and extreme irritability, which could be confused with manic symptoms. However, he has a history of a disturbance in premorbid functioning that predates the irritability by 18 months. He is also in the prime age range for new-onset schizophrenia, making this the more likely diagnosis.
24yo F with a diagnosis of bipolar disorder, manic, is treated with a mood stabilizer (lithium) and haloperidol, an antipsychotic. Which of these medications should be discontinued first once her condition has been stabilized?
Haloperidol should be d/c once stable b/c of risk of extrapyrimidal side effects (EPS)

Because tardive dyskinesia is irreversible, and the risk of its appearance increases with continued use, antipsychotics should be discontinued as soon as the patient’s psychotic symptoms remit, and not used again as long as the patient’s condition remains stable.
33yo F with bipolar disorder is 22 weeks pregnant. She has been taking valproic acid for her symptoms. Which of the following is the most likely abnormality that might be found on an ultrasound examination, due to the effects of the mood stabilizer?
Fetal spinal bifida

Maternal use of valproic acid is associated with a 1% to 2% risk of fetal neural tube defects such as spina bifida.
for a patient presenting in manic episode, how can you differentiate btw schizoaffective d/o and acute mania?
Schizoaffective disorder can be differentiated from acute mania by obtaining a longitudinal history and finding the presence of psychosis in the absence of mood symptoms, which confirms the former.
Repetitive behaviors or mental acts that a person feels driven to perform in response to an obsession according to a rigid set of rules. These behaviors or mental acts are aimed at preventing or reducing distress or preventing a feared event or situation. Typically, there is no realistic connection between the feared event or situation and the behavior or mental act.
COMPULSIONS
Recurrent and persistent thoughts or images that are experienced as intrusive and inappropriate and cause marked anxiety or distress. They are not simply excessive worries about real-life problems.
OBSESSIONS
A 17-year-old high school senior is referred to a psychiatrist by his counselor because of academic difficulty. Although he had always been an honors student, this past year his grades have quickly dropped, especially in mathematics. When questioned, he reveals the new onset of “superstitions” involving numbers. When presented with certain numbers, he feels compelled to count forwards and then backwards to and from that number. He becomes anxious about not completing this task, although he is unable to state a particular consequence. If interrupted, he must begin all over again. He realizes that there is “no good reason” for his behavior, but is unable to stop it. As a result of this, he not only feels “tortured,” but he may need to repeat this year in school. He denies any past psychiatric history. He had an appendectomy at age 15 and takes no medications. He does not drink alcohol or use tobacco products or illicit drugs. Which of the following treatments has demonstrated effectiveness in treating this condition?
individual & family CBT + pharmacotherapy

This individual suffers from OCD with his obsessions, compulsions, significant anxiety, and interference in his academic functioning. Whereas behavioral therapy and medications such as clomipramine and SSRIs are helpful in treating OCD, there is evidence that a combination of the two provides the greatest efficacy. There is an absence of studies documenting improvement in OCD solely with psychodynamic psychotherapy.
The patient in the previous question begins treatment for his condition. As treatment develops, the parents inform you that he had an episode of glomerulonephritis with antibiotics several weeks ago prior to developing these symptoms. What other possible issue might be related to his current symptoms?
Pediatric autoimmune neuropsychiatric disorders associated with streptococcal infections [PANDAS], have been specifically associated with development or exacerbation of OCD in children and adolescents. The symptoms will develop temporally after a demonstrated streptococcal infection.
The patient in the previous vignette is subsequently started on fluoxetine (Prozac, an SSRI). Which potential side effect is classified by the Food and Drug Administration (FDA) as a “Black Box Warning” issue?
suicidal ideations

This warning reads: “indicat- ing that antidepressants may increase the risk of suicidal thinking and behavior in some children and adolescents.”
A delirium characterized by disorientation, fluctuation in the level of consciousness, elevated vital signs, and tremors as the result of an abrupt reduction in or cessation of heavy alcohol use that has lasted for a prolonged period of time.
DELIRIUM TREMENS
Not actually a psychotic state but amnesia, both anterograde and retrograde amnesia, with confabulation that develops after chronic alcohol use. It is usually irreversible and is also caused by a thiamine deficiency.
KORSAKOFF SYNDROME (PSYCHOSIS)
An acute, usually reversible, encephalopathy resulting from a thiamine deficiency and characterized by the triad of delirium, ophthalmoplegia (typically sixth nerve), and ataxia.
WERNICKE SYNDROME
28yo M presents with a 12yr h/o regular alcohol use. Although he has been able to maintain employment as a truck driver, he often drives when “buzzed” in order to make his deadlines. He has been reprimanded on numerous occasions for failure to perform his job adequately, and this has led to increasing conflict with his wife. He denies any recent increase in drinking or any withdrawal symptoms, but he does admit to not getting “as drunk as I used to” following consumption of the same amount of alcohol. Which of the following factors in his history is the most specific for alcohol dependence?
not becoming as intoxicated as easily

Although DWI, marital conflicts, and occupational problems are all criteria for alcohol abuse, only not becoming intoxicated as easily as in the past is a criterion for alcohol dependence. Tolerance for alcohol (experiencing either the same effect with an increased amount or a decreased effect with the same amount), withdrawal from alcohol, and an inability to control alcohol use are the characteristics of alcohol dependence.
A 24-year-old woman states that her alcohol consumption is two glasses of wine or margaritas three times during the week and five vodka drinks on Friday and Saturday evenings. This pattern has been going on for nearly a year. She denies any impairment at work as a result of the drinking. She says she is “social” and goes out frequently. Further discussion reveals episodes of blacking out during drinking that have occurred five times in the previous 8 months. She describes these events as “having no recollection of leaving one bar and heading to the next.” Which of the CAGE questions is the most specific in screening this patient for alcohol dependence?
Have you ever attempted to Cut back on your drinking?

Although questions regarding the specific frequency, kind of alcohol, amount, and initial use of alcohol are important in establishing a history and pattern of abuse, only asking about attempts to cut back is specific for alcohol dependence, as this question addresses the inability to control alcohol use
48yo F is brought to the ED. She is unresponsive to questions, stumbles around the room, and is agitated. On physical examination, you notice that she smells of alcohol, and she is not cooperative during the remainder of the examination. Administration of what medicine would be the most appropriate initial treatment?
IV thiamine BEFORE glucose!

This patient presents with Wernicke encephalopathy, characterized by the triad of delirium, ataxia, and ophthalmoplegia. Thiamine must be given prior to glucose in patients suspected of having this disorder.
60yo M is brought to the ED by his wife for “confusion.” She reluctantly confides to the staff that he is a “heavy drinker,” that he had drunk up to a case of beer almost every day for the past 30 years. Although he has not changed his alcohol intake significantly, over the past year he has eaten less, preferring alcohol to large meals. She has noticed a gradual weight loss as a result. His last drink was earlier this day. Which of the following would be the most likely finding on the mental status examination of this patient?
confabulation

This patient has a long history of heavy, regular alcohol use and likely malnutrition. A common sequelae of this is chronic thiamine deficiency, resulting in Korsakoff syndrome. Korsakoff syndrome is characterized by an anterograde amnesia; this memory impairment is often (poorly) compensated for by the patient’s confabulation, or filling in the missing memories with false information.
A defense mechanism in which an individual deals with emotional conflict or stressors by refusing to acknowledge some painful aspect of external reality or subjective experience that is apparent to others. The term “psychotic denial” is used when there is gross impairment in reality testing.
DENIAL
A feeling that the world, or reality, has changed. The environment feels unreal or strange.
DEREALIZATION
A state of mood that is unpleasant, often sad.
DYSPHORIA
A defense mechanism in which an individual deals with emotional conflict or stressors by attributing exaggerated positive qualities to others. Use of this mechanism can alternate with devaluation, its opposite
IDEALIZATION
Thinking similar to that seen in young children in which the patient’s thoughts, words, or actions are seen to have power over external events. (For example: It snows in the winter because I buy sidewalk salt every fall.)
MAGICAL THINKING
What is an associated disorder in the family histories of patients with schizotypal personality disorder?
Schizophrenia

There is a greater association of cases of schizotypal personality disorder among biological relatives of patients with schizophrenia than among controls.
What feature must be present in a patient’s history for schizotypal personality disorder to be diagnosed?
The patient must have a history of cognitive and perceptual distortions.

The odd quality in which these patients perceive and think about the world is one of the diagnostic criteria for schizotypal personality disorder.
A 25-year-old man with schizotypal personality disorder comes to his psychiatrist with a chief complaint of a depressed mood. He notes that since losing his job as an astrologer, he has been depressed and unable to sleep. He states that although his mood is usually fairly low (4 out of a possible 10), it has lately been a constant 2. The patient also notes problems with concentration and energy level and has experienced several crying spells. He reports he had premonitions that certain foods could heal him, so he has been mixing “magical potions” and eating “magical foods.” A mental status examination reveals an oddly dressed man with constricted affect, ideas of reference, unusual beliefs, and some mild paranoia. Which of the following medications is most likely to be helpful to this patient?
Escitalopram (Lexapro) for depressive symptoms

Escitalopram is a selective serotonin reuptake inhibitor (SSRI) useful in the treatment of depression. Patients with schizotypal personality disorder who have either a depressive component to their illness or a secondary superimposed major depression (as is the case with this patient) should be treated with antidepressants.
A 50yo homeless veteran is brought to the ED by companions; he is not sure why he is there. He is clearly euphoric but also agitated and somewhat paranoid; he says he “feels fantastic” but is wary of answering any questions. On physical examination, the patient exhibits a moderately elevated blood pressure and pulse rate. He is most likely intoxicated with which of the following substances?
Cocaine intoxication (or other stimulants like amphetamine) can present with euphoria, irritability, anxiety, psychotic symptoms such as paranoia, as well as with elevated vital signs.

Conversely, intoxication with alcohol, barbiturates, benzodiazepines, and opiates generally causes depression, somnolence, and depressed vitals signs
What physical findings would be most likely present in the patients with cocaine intoxication?
answer: anorexia

Cocaine use causes decreased appetite and eventual weight loss. Like stimulants, cocaine intoxication can cause fever, insomnia, and mydriasis.
A cocaine addict is admitted to a detoxification unit in the hospital. After his euphoria and paranoia resolve, he is able to give a more complete history. He describes a 5-year history of almost daily crack cocaine use, with no periods of sobriety lasting for greater than several weeks. During these periods, he felt “depressed,” with an increased appetite, disrupted sleep, difficulty concentrating, and fatigue. He denies alcohol or other drug use, and his psychiatric review of systems is otherwise negative. What would be the most appropriate initial treatment for this patient?
narcotics anonymous

The most beneficial approach to cocaine addiction without additional psychopathology is to stress abstinence and relapse-prevention. Narcotics Anonymous meetings are easily accessible, approachable, and provide ongoing group and individual support. No medications have been proven to prevent cocaine cravings or relapse. While the patient admits to depressive symptoms when abstinent from cocaine, these symptoms are quite common (and self-limiting) during withdrawal. Also, the time period is not typical of a major depressive disorder. After 4 to 6 weeks of sobriety, the patient should be reevaluated for a depressive disorder with consideration to begin an antidepressant.
clinical pearls for cocaine intoxication
➤ Cocaine acts like a stimulant, causing euphoria, anxiety, increased energy/activity, and psychotic symptoms, as well as elevated vital signs, dilated pupils, weight loss, chest pain, and seizures.
➤ Denial is more the rule than the exception in regard to a patient’s aware- ness and acknowledgment of cocaine use.
➤ On average, cocaine is cleared from the body within 72 hours, and so a 3-day waiting period following its use is necessary for a patient to produce negative results from urine toxicology screening.
A 71-year-old woman with a history of early Alzheimer disease is hospitalized with pneumonia. During the course of the hospitalization, her family and primary physician notice a distinct worsening of her memory and alertness. What procedure would be the most sensitive in establishing a diagnosis of delirium?
answer: Electroencephalogram (EEG)

Although the other studies (CXR, EKG, CT brain, CBC) are all helpful in determining the etiology of delirium, only an EEG is sensitive in diagnosing this disorder. In almost all cases of delirium, an EEG shows generalized slowing. In cases where alcohol or sedative-hypnotic withdrawal is causing delirium, an EEG can show fast low-voltage activity. In hepatic encephalopathy, an EEG characteristically displays triphasic delta waves. Electroencephalogram findings typically remain normal early in the course of Alzheimer disease.
A 52-year-old man is hospitalized for triple CABG and subsequently develops a delirium. He has a history of one closed head injury with loss of consciousness, as well as past alcohol abuse with 7 years of sobriety under his belt. Which of the following factors is most likely the prime contributory factor in the development of his delirium?
A. His age
B. Hospitalization
C. Status post cardiac surgery
D. History of head injury
E. History of alcohol abuse
answer: s/p cardiac surgery

Advanced age is a major risk factor, with 60% of nursing home residents over age 75 experiencing repeated episodes of delirium. Of all medically ill, hospitalized patients, 10% to 30% exhibit delirium. However, some studies indicate that 90% of postcardiotomy patients experience delirium. Preexisting brain damage, prior history of delirium, alcohol dependence, diabetes, cancer, sensory impairment, and malnutrition may also contribute.
An 82-year-old man with a history of vascular dementia is brought to the hospital for increased agitation and urinary tract infection (UTI). Which feature most distinguishes effects of a delirium from dementia?
answer: altered level of consciousness

Both delirium and dementia can result in behavioral disturbances, cognitive deficits, and poor orientation. However, in all cases of delirium there is an alteration (reduction) in the level of consciousness, whereas in dementia (in the early stages) there is an alert, stable level of consciousness.
In the previous case, the patient is determined to have a delirium due to infection, overlying his dementia. Which of the following is the most important treatment approach in treating his delirium?
detection & correction of the underlying abnormality

Although environmental strategies and pharmacologic and physical interventions can be helpful and necessary to help orient patients or protect them from harm, the most essential treatment approach in all cases of delirium is to detect and correct the underlying cause of the disorder. The occurrence of an episode of delirium itself suggests a poor prognosis, meaning these patients have a significantly elevated future incidence of mortality. In the elderly, antipsychotic agents should be used with caution given some increased risk of mortality, and benzodiazepines may cause disinhibition, oversedation, or paradoxical excitation.
clinical pearls for delirium
➤ The hallmark of delirium is a fluctuation in the level of consciousness.
➤ Medications are a significant cause of delirium.
➤ An EEG is very sensitive in detecting delirium.
➤ The most important treatment approach for a patient with delirium is to detect and correct the underlying condition. Behavioral management can be accomplished with a low dose of a high potency antipsychotic or a short-acting benzodiazepine.
Loss of a subjective sense of pleasure.
ANHEDONIA
The content of the delusions or hallucinations reflects the nature of the illness. For example, in major depression, delusions and hallucinations are often about being defective,
deficient, diseased, or guilty and deserving of punishment.
MOOD-CONGRUENT DELUSIONS OR HALLUCINATIONS:
A syndrome characterized by hallucinations and/or delusions
(fixed, false beliefs). The individual’s ability to assess reality is impaired.
PSYCHOSIS
False beliefs about one’s body; in depression, these are
beliefs regarding illness, for example, that one has cancer and is about to die.
SOMATIC DELUSIONS
Symptoms of depression that are physiologic or are related to body functions, such as sleep, appetite, energy, and sexual interest.
Other symptom categories for depression are cognitive (poor concentration, low self-esteem) and emotional (crying spells).
VEGETATIVE SYMPTOMS:
A 10yo girl is brought in for treatment by her father following the death of her mother 6 weeks previously because of an unexpected heart attack. The father is worried because the child is not sleeping well, has lost 7 lb because of a decreased appetite, seems to be tired much of the time, and is preoccupied with memories of her mother. He notes that she cannot concentrate on her usual favorite television shows and has lost interest in many of her previous social activities. The patient reports that she deeply misses her mother, but she also smiles in recalling many pleasant memories of their life together. What is the most likely diagnosis?
Diagnosis: Normal bereavement.

This patient’s symptoms fulfill the criteria for major depression, but she is in the very early stages of bereavement when such behavior is considered normal. Her sleep problems are part of bereavement. If her symptoms do not diminish in the subsequent 4 months, the clinician should make a reassessment for major depression.
A 17yo honors student is brought to the ED by his parents. In the last academic quarter, his grades have suddenly dropped, he is irritable with friends and family, he has no energy, he doesn’t go to bed until 1 AM, and he has a poor appetite. He also has auditory hallucinations in which a man’s voice tells him that he is a “lazy bastard” and that his family “would be better off with him dead.” What would be the most appropriate initial pharmacologic treatment plan?
Antidepressant combined with antipsychotic medication

Lithium is an augmenting agent that can be considered later if necessary.
Psychotic depression is diagnosed in a 14-year-old boy, and he is treated with an antipsychotic agent, risperidone (risperdal), and an antidepressant. Three months later, his mood symptoms have resolved, and he is no longer psychotic. What's the next best step?
The antipsychotic medication should be discontinued via a taper.

The antipsychotic agent should be discontinued because the psychotic symptoms have abated. The antidepressant should be continued for ~6 to 9 months in a patient with the first onset of major depression, and for longer (perhaps indefinitely) for a patient with recurrent depression.
Clinical Pearls for major depression with psychotic features
➤ Psychotic symptoms indicate severe depression and should prompt a serious consideration of hospitalization.
➤ Children and adolescents with major depression often report their mood as angry or mad as opposed to sad or depressed.
➤ Patients with depression are relieved when the clinician inquires about suicidality; asking about suicidal thoughts does not increase the risk that patient will kill themselves. Suicide rates in 15- to 19-year-olds have quadrupled over the last four decades and are usually in the top three or four causes of death in adolescents. Always ask about suicide in depressed children and adolescents (Have you ever wanted to die? Have you ever thought about or tried to kill yourself?).
➤ According to CMAP, youth suffering from major depression with psychosis should initially be started on an SSRI and an atypical neuroleptic with doses of both increased gradually until a therapeutic response is achieved, a maximum dose for body size is reached with no response, or the patient is experiencing side effects.
➤ If the child suffering from major depression with psychotic features responds to the combination of antidepressant and atypical antipsychotic, the atypical antipsychotic should be continued for 3 months and then tapered off. The antidepressant should be continued for 6 to 9 months and then tapered over 2 to 3 months by 33% per month.
A 15yo girl gets into fights, intimidates others, steals, skips school (resulting in falling grades), and breaks her curfew regularly. She does not exhibit any remorse for her behavior. She denies having any depressive symptoms such as sleep or appetite disturbances and says she feels pretty good about herself. She does not report any suicidal or homicidal thoughts. Most likely dx? Tx?
Most likely diagnosis: Conduct disorder (CD)

➤ Best treatment:Multisystemic treatment(MST) approach with involvement of parents and teachers. The treatment of CD can be difficult. There are few studies that look at CD systematically and study various treatment techniques. Over the last few years, a number of new studies have begun to look at how to treat CD. In terms of behavioral interventions, multisystemic-based therapeutic approaches are quite helpful. These approaches combine a well-coordinated plan to help parents develop new skills at home, such as parent-child interaction training, to help the relationship between parents/caregivers and the child. In addition, it is helpful to teach classroom social skills, institute playground behavior programs, and facilitate and encourage communication between teachers and parents. Psychopharmacologic interventions also show some promise. Many children with CD have a comorbid diagnosis of attention- deficit/hyperactivity disorder (ADHD). This needs to be identified and treated. Even if a child does not meet full diagnostic criteria for ADHD, there is evidence that CD is amenable to treatment with stimulants— often leading to less aggression and impulsiveness. There are also a few studies showing that the use of newer antipsychotics, such as risperi- done, may be useful with CD.
A 14yo girl’s family brings her to treatment against her wishes. The family reports she has a very short temper, argues with them and teachers at school daily, often refuses requests at both settings, and usually blames her teachers or her parents for the behaviors she is showing. She denies being depressed, anxious or any psychotic symptoms. With this information, what might be the best diagnosis?
oppositional defiant disorder

The patient does not yet meet criteria for conduct disorder, which might include additional signs such as aggression to people or animals, destruction of property, theft, or significant rule violation. Both conditions cause impairment in social, family, and academic functioning.
Over the next 2 years, this patient (14yo with oppositional defiant disorder) continues to struggle and is arrested for truancy, fighting, and possession of drug paraphernalia. Given this information, what might we best be able to say about patients with this diagnosis in general?
answer: Patients are more likely to have parents who have antisocial personality disorder and alcohol dependence.

This patient now fits the criteria for conduct disorder. Conduct disorder is more common in children of parents with antisocial personality disorder and alcohol dependence than it is in the general population.
What treatment is best to treat the comorbid depressive symptoms of an adolescent with Conduct Disorder?
antidepressant medication

If the criteria for a comorbid condition are met, this disorder should be the first target of psychopharmacologic intervention.
Clinical Pearls of Conduct Disorder
➤ Conduct disorder can be a predecessor of antisocial personality disorder.
➤ Oppositional defiant disorder can be a predecessor to conduct disorder
➤ Treatment of CD is very difficult and typically involves a community-oriented and highly organized treatment approach.
➤ When there are comorbid psychiatric conditions, they should be the target of psychopharmacologic interventions.
➤ Conduct disorder can be diagnosed prior to age 18, whereas antisocial disorder cannot be diagnosed until after age 18.
A defense mechanism by which an individual deals with emotional conflict or stressors with words or behavior designed to negate or to symbolically make amends for unacceptable thoughts, feelings, or actions. This mechanism can be realistically or magically associated with the conflict and serves to reduce anxiety and control the underlying impulse.

An example is seen in the child’s game in which one avoids stepping on cracks in the sidewalk to avoid “breaking your mother’s back.”
Undoing
A defense mechanism by which an individual deals with emotional conflict or stressors by concealing the true motivations for thoughts, actions, or feelings through the elaboration of reassuring or self serving but incorrect explanations. For example, a woman steals a coat from a local department store although she can afford to pay for it. She tells herself, “It’s okay—that department store has plenty of money, and they won’t miss one coat!”
RATIONALIZATION:
A defense mechanism by which an individual deals with emotional conflict or stressors by separating ideas from the feelings originally associated with them. The individual loses touch with the feelings associated with the given idea (eg, the traumatic event) although remaining aware of the cognitive elements of it (eg, descriptive details). For example, a man comes home to find his wife in bed with another man. Later, describing the scene to a friend, the man can relate specific details of the scene but appears emotionally unmoved by the whole event.
ISOLATION OF AFFECT:
A 24yo F is called into the head office of the agency where she works and told that her chronic lateness in completing her assignments will result in her dismissal if she does not change her behavior. The patient really loves her job, and the news comes as a major blow. That night at home, she tells her boyfriend in great detail about each and every step of the meeting and spends the entire night thinking about her job. The boyfriend tells her that she does not “look” particularly upset. Which of the following defense mechanisms is being used by this woman?
Intellectualization is a defense mechanism by which an individual deals with emotional conflict or stressors with an excessive use of abstract thinking to control or minimize disturbing feelings. Because the stressors have been successfully defended against in this instance, the patient does not appear particularly distressed.
A 23yo medical student makes lists of all the tasks that he must accomplish each day. He spends hours studying and refuses to go out with his colleagues even when there are no tests on the immediate horizon, preferring to spend his time looking at specimens in the laboratory. He keeps meticulous notes during all his classes and prefers to attend every lecture, not trusting his colleagues to take notes for him. He is doing well in school and has a girlfriend who is also a medical student. Which disorder does this student most likely have?
Obsessive-compulsive traits

Although this student clearly demonstrates some traits of obsessive- compulsive behavior, his social and occupational functioning are both good, which rules out the personality disorder.
A 26yo F comes to see a psychiatrist because she has been taking showers for 6 to 7 hours every day. She explains, “It all starts when I wake up. I am sure I am covered in germs, and if I don’t wash, I will get sick. If I don’t wash, I get paralyzed with anxiety. Once I’m in the shower, I have to shower in a particular order. If I mess up, I have to start over, and this takes hours and hours. My skin is cracking and bleeding because I spend so much time in the water.” What disorder does this patient most likely have?
This patient demonstrates the classic obsessions, followed by compulsions, of OCD.
For the patient with OCD, what treatment is best?
Cognitive-behavioral therapy (CBT)-evoked response prevention

The standard pharmacologic approach to the treatment of OCD is to prescribe a selective serotonin reuptake inhibitor (SSRI) or clomipramine, although these are not answer choices. The best psychotherapeutic choice would involve gradually exposing the patient to the anxiety- provoking circumstance and teaching her how to manage that anxiety through CBT techniques.
Clinical Pearls for OCD
➤ Patients with obsessive-compulsive personality disorder are characterized by their rigidity, stubbornness, and perfectionism so that they often have trouble meeting deadlines at work or making choices. They tend to be work-centered to the exclusion of enjoying social activities and leisure time. They are often miserly with money and hoard possessions exces- sively. They do not exhibit frank obsessions and compulsions.

➤ Physicians can use the preoccupation with rules and order shown by these patients to teach them to self-monitor their own conditions. These indi- viduals can be extremely compliant.They need to know the details of their condition in scientific language.

➤ Patients with OCD have prominent obsessions and compulsions that alter- nately create anxiety and reduce it (through the compulsive behavior).
➤ Patients with obsessive-compulsive personality disorder have pervasive
patterns of behavior that include rigidity and perfectionism but not true
obsessions and compulsions.
➤ Patients with obsessive-compulsive personality traits often resemble
patients with the personality disorder. The difference is one of degree and impairment of function. Individuals who are significantly impaired can exhibit symptoms that meet the requirements for the personality disorder.

➤ Defense mechanisms include rationalization, intellectualization, undoing, isolation of affect, and displacement.
A 34-year-old woman suffered a traumatic event 1 year ago. Since that time, she has been depressed, irritable, angry, and disconnected emotionally. She has trouble sleeping and concentrating. She has nightmares about the rape, tries not to think about it, and avoids going near the place where it occurred. On mental status examination she shows a depressed mood that is congruent with her affect, which is also restricted. She has violent fantasies toward her attacker but no overt homicidal or suicidal ideation.
➤ What is the most likely diagnosis?
➤ Should this patient be hospitalized?
➤ Most likely diagnosis: Posttraumatic stress disorder (PTSD).
➤ Should this patient be hospitalized: No. Although she has passive homicidal ideation (which is fairly typical in this kind of circumstance), she has no specific intent or plan to cause “something terrible to happen” and does not know her attacker or his location. This patient is not committable. Admission to the hospital should not be offered on a voluntary basis either, as she would probably do well on an outpatient basis.
A 36yo businessman who survived a serious car accident 4 months ago complains of “jitteriness” when driving to work and is currently using public transportation because of his anxiety. He has found himself “spacing out” for several minutes at a time at work and having difficulty concentrating on his job. He has trouble sleeping at night, has lost 4 lb because of a decreased appetite, and admits that his job performance is slipping. What is the most likely diagnosis?
dx: major depression

This patient exhibits the features of major depression, which com- monly occurs with PTSD as a comorbid condition. Also, this patient exhibits two of the three symptom clusters for PTSD, and a clearly traumatic event has preceded his symptoms. It would be unlikely for him to suddenly develop an anxiety disorder such as panic disorder, social phobia, or specific phobia. While the “spacing-out” periods are likely to be episodes of dissociation occurring as a result of the trauma, possible neurologic injury should be considered, especially because of his history and the change in his job performance. In addition, the patient might be using alcohol to aid in sleeping or to decrease the hypervigilance he has experienced since the accident.
Whats the most efficacious treatment for PTSD?
Cognitive-behavioral therapy (CBT)

While different varieties of psychotherapy have been found to be effective in PTSD, various forms of cognitive-behavioral therapy have been the best studied and most useful.

Dialectical-behavioral therapy is a specific therapy developed for the treatment of borderline personality disorder.
Despite a course of psychotherapy, the patient with PTSD continues to suffer from recurrent nightmares, flashbacks, hypervigilance, and emotional numbing. What medication is most likely to be helpful in this patient?
Prazosin

Individuals with PTSD often respond to SSRIs. Additionally, alpha-2 agonists such as prazosin have also demonstrated efficacy in significantly reducing symptoms of PTSD. However, buspirone, second-generation (“atypical”) antipsychotics (such as risperidone), benzodiazepines, and mood stabilizers (such as valproic acid) are not recommended as monotherapy for the treatment of PTSD. Although alprazolam might assist in decreasing the patient’s general anxiety, the incidence of sub- stance abuse is high among patients with PTSD; thus, addictive medications should be avoided in these individuals.
A 34-year-old man suffered from major depression in the past and, according to his history, at least a 10-year-period of depressed mood with insomnia, a fluctuating appetite, and a decreased ability to concentrate. He also notes that his self-esteem is low. He is experiencing no suicidal ideation, psychotic symptoms, or weight loss and is able to continue working. He denies any other psychiatric symptoms or medical problems.

Most likely dx?
tx?
➤ Most likely diagnosis: Dysthymic disorder.

➤ Best medical therapy: Selective serotonin reuptake inhibitors (SSRIs), selective norepinephrine reuptake inhibitors (SNRIs) as well as other antidepressants such as bupropion can be helpful in many patients with this disorder. Although other antidepressants such as tricyclic antidepressants (TCAs) and monoamine oxidase inhibitors (MAOIs) can be effective, SSRIs and SNRIs have better side effect profiles and are usually the first choice.

This patient has at least a 10-year history of a depressed mood; this duration fulfills the 2-year requirement for a diagnosis of dysthymic disorder. Although he experiences a fluctuating appetite and insomnia, neither appears to be severe. (The patient is able to continue to work and has not lost any weight.) He complains of other symptoms consistent with dysthymic disorder, such as poor concentration and low self-esteem. He does not have psychotic symptoms or suicidal ideation, either of which suggest a more severe disorder. He experienced a major depression in the past, but does not currently meet criteria, and his prior episode did not occur during the first 2 years of the dysthymic disorder. The patient denies alcohol, drug abuse, or medical problems, all of which can mimic dysthymic disorder; however, a complete history, physical examination, and laboratory studies should be performed.
A 22-year-old woman is referred to your office by her family physician for evaluation of “depression.” Her primary care doctor is unsure whether she is suffering from dysthymic disorder or a major depressive disorder. Which of the following characteristics is more consistent with dysthymic disorder versus major depression?
Symptoms are of a low intensity

Although the distinction between dysthymic disorder and major depressive disorder can sometimes be challenging (especially if the major depressive illness is chronic and/or recurrent), patients with dysthymic disorder tend to have an earlier onset of lower intensity symptoms, a more chronic course, fewer neurovegetative symptoms, lack of psychosis, and less severe psychosocial or occupational impairment when compared to individuals with major depression.
The previous patient is evaluated fully and determined to have dysthymic disorder. Which of the following medications is the most appropriate first-line treatment for her?
Sertraline (Zoloft)

SSRIs (such as sertraline), SNRIs, and bupropion have demonstrated efficacy in treating dysthymia. Although TCAs and MAOIs are also beneficial, newer antidepressants such as SSRIs or SNRIs are better tolerated and safer in overdose. Neither lithium nor lorazepam is indicated for dysthymic disorder.
the previous patient returns to your office for a medication check. Sertraline was started 4 months ago. The dose has been increased twice and the patient has been taking 200 mg for 2 months. She does not feel the medication has provided any relief to her symptoms, but she has tolerated the medication well. What would be the next step for this patient?
d/c and start another SSRI

SSRIs are generally tolerated better by patients and are often beneficial in treating dysthmia. Two hundred milligrams of sertraline is the maximum dose and 6 to 10 weeks is considered an adequate trial, so another SSRI should be tried before switching to another class. Augmentation is usually done when the patient has a partial response to an initial medication.
clinical pearls for dysthymic disorder
➤ Patients with dysthymic disorder can function relatively well in their lives but experience subjective symptoms of a depressed mood and mild vegetative symptoms.
➤ Dysthymic disorder can be diagnosed in children if they are symptomatic over a 1-year time period (instead of the 2 years required for adults).
➤ Dysthymic disorder can be successfully treated with antidepressant medication, psychotherapy, or a combination of the two.
A 69-year-old man with a history of high blood pressure has been having problems with memory and disorganized behavior for the last few months. Previously, he functioned at a much higher level than he currently does. Mild aphasia and memory impairment are noted on a mental status examination.

Most likely dx?
next step?
➤ Most likely diagnosis: Dementia.
➤ Next step: Reversible causes of dementia should be ruled out.

The patient is a reasonably healthy elderly gentleman. He is experiencing an insidious onset of problems with his memory and his organizational ability, which worsened markedly over the past few months. Previously (according to his history), the patient functioned at a much higher level. Mild aphasia and memory impairment, noted on the mental status examination, lend further credence to the diagnosis.
A 75-year-old man is brought in by his daughter to a psychiatrist for an evaluation. He has become increasingly forgetful over the past year, missing engagements with his children and grandchildren. He is also unable to remember directions, resulting in his becoming lost when driving alone. He has no psychiatric history, although his wife died 14 months ago. His medical history is significant for poorly controlled hypertension. Which of the following additional features is necessary in order to accurately diagnose dementia?

A. Agitation
B. Fluctuation in consciousness
C. Radiographic findings
D. Hallucinations
E. Another cognitive deficit
another cognitive deficit

For a diagnosis of dementia, one or more additional cognitive deficits must be present in addition to memory impairment. They can include aphasia, apraxia, agnosia, or a disturbance in executive functioning. Individuals with vascular dementia can demonstrate findings on a CT or MRI scan, such as lacunar infarcts or evidence of past strokes. Patients with Alzheimer disease may demonstrate generalized cortical atrophy and ventricular enlargement. Individuals with dementia usually remain alert, whereas those with delirium display a fluctuation in consciousness. While psychotic symptoms such as delusions and hallucinations, as well as agitation, can be seen they are neither specific to nor necessary for the diagnosis of dementia.
Given the loss of a spouse suffered by the patient in previous question, a depressive disorder (pseudodementia) as a cause of his memory problems is considered in his differential diagnosis. How would this patient be predicted to perform on cognitive testing if he has a dementia rather than a depressive illness?
Better effort with poor insight

On cognitive testing, patients with dementia generally put forth considerable effort, display poor insight into their deficits, and minimize their problems. In contrast, those with depressive disorders often are apathetic, make little effort, but complain about their memory problems.
An MRI of the brain is performed for the patient in previous question. It demonstrates numerous small infarcts throughout the deep white matter structures. Which of the following best describes the likely progression of his illness?
Stepwise deterioration

Given his history of hypertension and lacunar white-matter infarcts, his diagnosis is most likely vascular (multiinfarct) dementia. Vascular dementia characteristically demonstrates a stepwise deterioration in cognitive functioning corresponding to either small ischemic events or emboli causing strokes. Alzheimer disease progresses gradually over many years, whereas dementias caused by head trauma or anoxia are generally sudden in onset and have a subsequently stable course. Dementias with certain infectious etiologies, such as Creutzfeldt–Jakob disease, have a fairly rapid rate of progression.
As the dementia in the patient in previous qeustion progresses further, he becomes more agitated and aggressive. Which of the following treatment options would be the most appropriate for his behavior?
High-potency antipsychotic

Treatment with a low dose of a high-potency antipsychotic is an appropriate pharmacologic approach to decreasing agitation and aggression in individuals with dementia. Lower-potency antipsychotics should be avoided given their significant anticholinergic and orthostatic side effects. Short-acting benzodiazepines can also be used but may disinhibit patients and worsen their behavior. Long-acting benzodiazepines should be avoided because they can build up in elderly patients and cause oversedation or ataxia. Antidepressants can be useful in treating the depressive and neurovegetative symptoms that are common in dementia, but they would not address the agitation or hostility. Acetylcholinesterase inhibitors can be helpful in improving cognition in the early and mid stages of Alzheimer disease.
clinical pearls for dementia
➤ The diagnosis of dementia requires both memory impairment and another cognitive deficit.

➤ The MMSE is a very sensitive screening tool for diagnosing dementia.

➤ Alzheimer disease is the most common form of dementia; vascular diseaseis the next most common.

➤ Most dementias have a slow rate of onset, are irreversible, and are
progressive.

➤ Either a low-dose of a high-potency typical or atypical antipsychotic or a
low-dose, short-acting benzodiazepine can be helpful in decreasing the agitation seen in patients with dementia.
A 54-year-old woman is sent to a psychiatrist by her physician. She complains of numerous vague symptoms and believes that they are caused by a serious illness. These problems resulted in her scheduling numerous appointments with the physician in the past year. She is concerned about the prospect of having a serious disease, although she is temporarily relieved when test results come back negative. Her behavior has serious ramifications in her work life.

dx?
tx?
➤ Most likely diagnosis: Hypochondriasis.

➤ Best approach: Schedule frequent, regular clinic appointments with the patient to assure her that her complaints are being taken seriously. The goal is to have contact with the patient before the relief she gets from reassurance fades to the point where she is convinced she has a new disease. Invasive diagnostic techniques or procedures should be avoided unless there is convincing objective evidence that they are necessary. The patient is unlikely to follow up with referrals to a psychiatrist or other mental health professional. The primary care physician is going to have to do the supportive counseling.
A 42-year-old woman describes a 20-year history of numerous physical complaints, including joint pain, dysuria, headaches, chest pain, nausea, vomiting, irregular menses, and double vision. Although they do not all occur at the same time, she has been suffering from one or more of these problems throughout her adult life. Many workups have been done for her, and she has undergone repeated hospitalizations, but no specific cause has yet been found. She is extremely anxious and has become significantly disabled as a result. Which of the following is the most likely diagnosis?
somatization disorder

The most likely diagnosis for this woman is somatization disorder. She presents with numerous somatic complaints, related to several bodily areas, which are not fully explained by a medical cause. The focus is on the symptoms themselves, not on a perceived physical defect (as in body dysmorphic disorder), on the fear of having a specific disease (as in hypochondriasis), or on symptoms of pain (as in pain disorder).
A 26-year-old woman presents to her physician with the chief complaint of, “I have epilepsy.” She states that for the past 3 weeks she has had seizures almost daily. She describes the episodes as falling on the ground, followed by shaking her arms and legs uncontrollably. These events last for approximately 10 minutes. She is unable to otherwise move during the time, although she denies any loss of consciousness, bladder, or bowel functions. She has never injured herself during these, but as a result she has been unable to continue her job. She is somewhat bothered as she received a promotion 1 month ago. Which of the following is the most likely diagnosis?
conversion disorder

The most likely diagnosis in this woman is conversion disorder (“pseudo seizures”). Conversion disorder patients present with neurological symptoms (eg, sensory deficit, motor weakness, seizures) that are felt to be unconsciously produced and believed to be caused by a psychological conflict or stressor. It is unlikely a seizure disorder given her retention of consciousness and lack of incontinence or injury. Her focus is not on an imagined defect in appearance, on the fear of having a serious illness caused by misperceived body sensations, or on multiple physical complaints.
A 36-year-old male is referred to a primary care physician for evaluation of his complaints. He is convinced that he has colon cancer despite being told that it is unlikely because of his young age. He occasionally notices blood on the toilet paper and has abdominal cramps when he eats too much. A review of the records demonstrates numer- ous prior appointments in connection with the same or similar complaints, including repeatedly negative results from tests for occult fecal blood and normal results from colonoscopies. He continues to be worried about dying of cancer and requests another colonoscopy. Which of the following is the most likely diagnosis?
hypochondriasis

The most likely diagnosis for this man is hypochondriasis. His chief complaint is a concern that he has colon cancer. He remains focused on this illness despite prior evaluations with negative results and reassurance from his physician. Although he has several gastrointestinal symptoms (blood in stools and abdominal cramps), he is probably misinterpreting them. His worry is caused by fears of having colon cancer, not about a distorted body image, pain sensations, or numerous physical symptoms.
Which of the following strategies by the primary care doctor would be the most effective in treating the patient in the previous question (hypochondriasis)?
regularly scheduled appointments with reassurance

The most effective strategy for treating individuals with hypochondriasis is to schedule regular appointments. In this way, any physical complaints are addressed, and reassurance is provided, albeit temporarily. This approach also minimizes both doctor shopping and unnecessary testing. Treatment with an antianxiety (or antidepressant) agent is not helpful in hypochondriasis, unless a comorbid anxiety (or depressive) disorder is present. Because individuals with this disorder are fearful of having a medical illness, they usually resist seeing a psychiatrist.
clinical pearls for hypochondriasis
➤ The distinctive feature of hypochondriasis is a fear of having a serious illness based on a misinterpretation of bodily sensations.

➤ Extensive, repetitive, or invasive tests or procedures should be avoided in individuals with hypochondriasis unless a clear clinical indication is present.

➤ Antidepressants and antianxiety medications are not indicated unless
comorbid depressive or anxiety disorders are present.

➤ The most effective treatment for patients with hypochondriasis is to
schedule frequent, regular appointments with the same primary care physician, coupled with education and reassurance.
A 32-year-old man who is incarcerated is seen by a psychiatrist after he gets into a fight with a fellow inmate. He has a long history of incarcerations and an inability to conform to societal norms, which appears to have begun as early as age 13. He does not seem to be remorseful about his actions.

➤ What is the most likely diagnosis?
➤ What other psychiatric disorders might one suspect in this patient?
➤ Most likely diagnosis: Antisocial personality disorder.

➤ Other suspected psychiatric disorders: High on the list of comorbid psychiatric disorders is any substance use disorder.
A defense mechanism in which an individual guards against uncomfortable emotional conflicts or stressors through actions rather than reflections or feelings. For example, a man who has a hard day at work and is humiliated by his boss goes out that night and starts a fight at a local bar.
ACTING OUT
A defense mechanism that helps an individual deal with emotional conflict or stressors by falsely attributing to another person the individual’s own unacceptable feelings, impulses, or thoughts. Not infrequently, the individual induces in others the very feelings that he or she first mistakenly believed to be there, thus setting into motion a self-fulfilling prophecy. For example, a paranoid patient has uncomfortable hostile emotions and projects them onto another person, believing that the other is hostile to him. He then behaves as if the other person is going to be hostile to him. The other person, seeing the suspiciousness and the withdrawal of the patient, eventually acts in a frustrated (or seemingly hostile) manner toward the patient, thus completing the loop.
PROJECTIVE IDENTIFICATION
A 39-year-old man is evaluated by mental health services in prison. He has a history of multiple arrests as both an adult and as a juvenile. After several interviews, a diagnosis of antisocial personality disorder is confirmed. He has a history of multiple psychiatric hospitalizations after suicide attempts, and was in special education programming as a child. Which psychiatric diagnosis is most likely to have occurred comorbidly in such an individual?
conduct disorder

Although all the diagnoses listed are frequently comorbid, evidence of a diagnosis of conduct disorder with onset before age 15 is required for a diagnosis of antisocial personality disorder.
A 16-year-old adolescent female is incarcerated in a juvenile detention facility. She is currently charged with theft, apparently to support her and her boyfriend’s drug habit. She has had multiple involvements with child and family services for running away from home, where she apparently had been sexually abused by her mother’s boyfriend. She has a diagnosis of posttraumatic stress disorder (PTSD). Prior to the onset of the abuse, she was doing extremely well in school, in an accelerated program. Which of the following factors speaks most strongly against a diagnosis of antisocial personality disorder?
her age

Personality disorders cannot be diagnosed before 18 years of age. Antisocial personality disorder should not be overlooked in females, even though it is much more common in males. Antisocial actions committed solely during psychotic or manic episodes, or to support a drug habit, would not support a diagnosis of antisocial personality disorder.
A 39-year-old man with antisocial personality disorder, incarcerated for life after murdering a man, has a multitude of somatic complaints over the course of several years. Yearly physical examinations never show anything physically wrong with him, yet he complains of a variety of aches and pains, neurologic symptoms, and gastrointestinal distress. He does not enjoy the time he spends in the jail’s infirmary. Which of the following is the most likely explanation for this patient’s complaints?
he has developed a somatization disorder

Development of a somatization disorder becomes more common in patients with antisocial personality disorder as they grow older. There is no evidence of secondary gain here (which rules out malingering), nor is there evidence of psychotic thinking. Physical examinations have all been negative (making a physical illness less likely) and the complaints are all around somatic symptoms, making a pure anxiety disorder less likely as well.
clinical pearls for antisocial personality disorder
➤ Patients with antisocial personality disorder show a reckless disregard for, and violation of the rights of others starting in childhood or adolescence. The disorder itself cannot be diagnosed until the patient is 18 years old, although evidence of conduct disorder must be seen before the age of 15.

➤ Physicians should adopt a firm, no-nonsense, yet nonpunitive approach with these patients.

➤ Substance abuse disorders are often comorbid in patients with antisocial personality disorders. However, if the antisocial behavior occurs only in context of behavior designed to procure drugs and never when the patient is sober, the personality disorder should not be diagnosed.

➤ Patients with antisocial personality disorder evidence it in all facets of their lives, not just, for example, when robbing a store for personal gain. Unlawful behaviors have many other etiologies other than antisocial personality disorder.
➤ Defense mechanisms commonly used in patients with antisocial personality disorder are projective identification and acting out.
15-year-old girl with past diagnosis of and evidence of major depression was recently hospitalized for a suicide attempt. She is treated for this and seems to respond well. The parents however note that she has some evidence of paranoia that is still present after the mood symptoms have resolved.

➤ What is the most likely diagnosis?
➤ What is the best therapy for this condition?
➤ Should this patient be hospitalized?
➤ Most likely diagnosis: Schizoaffective disorder

➤ Best therapy: Anantipsychotic agent (such as haloperidol or risperidone) should be tried initially. If it is ineffective alone, antidepressants (a selective serotonin reuptake inhibitor [SSRI] is generally tried first) should also be administered.

➤ Is hospitalization needed: No. The patient is currently not a danger to herself or others and appears to be able to care for herself. This disorder as presented here should be treated in an outpatient setting unless the suicidal ideation returns and/or worsens.
Lack of energy
ANERGIA
Lack of interest in one’s usual pleasure-seeking activities, such
as hobbies.
ANHEDONIA
A 17-year-old adolescent female is seen in your office after her friends noticed some strange behavior. The patient reports to you that in addition to some long-term depression issues, she has begun to experience some other disturbing events. She reports that over the last 2 months she has been hearing voices—both at work and at home—of people who she does not think are there. She doesn’t recognize these voices. Sometimes they just give a dialogue of what she is doing, but, more disturbing to her is when they start saying horrible things about her and tell her to do things she does not want to do. You start her on olanzapine and she returns in 1 week and the voices have gone entirely for 2 or 3 days. However, she continues to experience severe mood symptoms. Her Hamilton Depression rating scale score places her in the moderate to severe range for depression. What should you do next?

A. Inform the patient that these symptoms are the negative symptoms common to the disorder.
B. Refer the patient for supportive psychotherapy.
C. Treat the patient with fluoxetine (an SSRI).
D. Increase the dose of the antipsychotic.
E. Add a mood stabilizer to the regimen.
treat the patient with fluoxetine (SSRI)

Although data are not clear as to the efficacy of administering antidepressants to patients with schizoaffective disorder (and depressive symptoms), the continued presence of depressive symptoms makes this treatment worth trying.
A 28-year-old man is brought to a psychiatrist complaining that he has been hearing voices for the past several weeks. He says that he also heard these voices 3 years ago. He notes that his mood is “depressed” and rates it 3 on a scale of 1 to 10 (with 10 being the best he has ever felt). He does not recall if his mood was depressed the last time he had psychotic symptoms. Which of the following actions should the physician take next?
Obtain more detailed information about the time course of the
psychotic symptoms and the mood symptoms (before started any drugs - other choices were to start an antipsychotic or antidepressant)

The time course of the mood symptoms and psychotic symptoms determines the treatment of the patient because the diagnosis can be schizoaffective disorder versus major depression. Although the patient should undergo a urine toxicology screening, this should not be done until a complete history is obtained so that further targeting of laboratory testing can be accomplished.
A 40-year-old man with schizoaffective disorder has been hospitalized in an inpatient psychiatry unit for the third time in the last 5 years. During each episode, he becomes noncompliant in taking his medications, develops manic symptoms and auditory hallucinations, and then becomes violent. In the inpatient unit, he physically threatens other patients and staff and is generally agitated. He is put in isolation to help quiet him. The patient is prescribed a mood stabilizer and an antipsychotic medication. Which of the following medications might also help relieve this patient’s acute agitation?
lorazepam (Ativan)

Benzodiazepines such as lorazepam (Ativan) and clonazepam (klonopin) have been shown to be effective adjunctive treatments for acute mania both in patients with schizoaffective disorder and in patients with bipolar disorder.
clinical pearls for schizoaffective disorder
➤ Unlike patients with schizophrenia, patients with schizoaffective disorder have mood symptoms that occur during significant portions of their illness.
➤ Once a very clear longitudinal history of symptoms and functioning is obtained, it is often possible to diagnose either a bipolar illness or schizo-
phrenia in a patient with schizoaffective disorder.
➤ Patients with schizoaffective disorder and manic mood symptoms should be treated with a mood stabilizer and an antipsychotic. Patients with schizoaffective disorder and depressive mood symptoms should be treated with an antipsychotic alone; if this is not effective, an antidepressant should also be used.

➤ Younger patients with schizoaffective disorder may exhibit more severe symptomatology than adult patients
A 24-year-old woman has olfactory hallucinations several times a week for the past month. She smells burning rubber and rotting flesh. Her friends report seeing her “staring off in space,” although she herself is unaware of this behavior. The patient has no other psychiatric symptoms. She has a history of a concussion resulting from a car accident 1 year previously.

➤ What is the likely diagnosis for this patient?

➤ What further diagnostic workup would establish the diagnosis?

➤ On what axis based on the Diagnostic and Statistical Manual of Mental Disorders, 4th edition (DSM-IV) are medical problems coded?
➤ Most likely diagnosis: Psychosis secondary to a general medical condition. The medical condition here is most likely a partial complex seizure disorder.

➤ Diagnostic test: An electroencephalogram(EEG) or imaging of the brain using computed tomography (CT) or magnetic resonance imaging (MRI).

➤ Axis that medical problems are coded: Psychosis secondary to a general medical condition is coded on axis I, but the seizure disorder itself is coded on axis III.

Patients with new-onset psychotic symptoms, particularly olfactory, and gustatory symptoms, should have medical conditions ruled out first. One year after experiencing head trauma in which she lost consciousness and suffered a concussion, this patient presents with new-onset olfactory hallucinations. In addition, according to her friends, she has spells of “zoning out,” which might point to the occurrence of a seizure. The facts that the patient has no other psychotic symptoms, is aware that no one else smells what she smells, and otherwise feels and behaves normally all point away from a major psychotic disorder such as schizophrenia. As a general rule, olfactory and tactile hallucinations are more common with medical conditions rather than psychosis.
A hallucinated sensation that insects or snakes are crawling over the skin. It is a common side effect of extensive use of cocaine or amphetamines.
FORMICATION
A 47-year-old man with hemophilia presents to his physician with new onset symptoms including hallucinations (he hears his dead grandmother calling his name). He has no prior psychiatric history. He realizes the symptoms are odd, but he is not overly disturbed by them. He is diagnosed with HIV/AIDS. Which of the following management strategies should be tried first?
try to treat HIV/AIDs aggressively

Psychosis due to a general medical condition first and foremost should be treated by identifying and vigorously treating the underlying medical condition. Since this patient’s psychotic symptoms don’t bother him tremendously, and he is not behaving in any way that could be dangerous to himself or others, treatment with an antipsychotic is not necessary first line. If such a treatment became necessary (if the reduction in viral load, etc., does not terminate the hallucinations), then treatment with a high potency atypical antipsychotic medication should be tried next. Psychodynamic psychotherapy would not be useful in this scenario.
A 30-year-old woman is brought to her physician by her brother, who states that the woman has been having numerous psychiatric symptoms. These include hearing voices, seeing “ghosts,” and tasting a sour taste even when she hasn’t eaten anything. On mental status examination by the physician, the patient is also noted to have problems with her reality testing and loose associations. Which of the patient’s symptoms are most indicative of a psychosis secondary to a general medical condition?
gustatory hallucinations

Auditory and visual hallucinations are common in all episodes of psychosis regardless of the cause. Gustatory hallucinations (along with olfactory and tactile hallucinations) can be more common in psychoses caused by a medical illness. It would be somewhat unusual to find problems with reality testing or thought disorders in a patient with psychosis due to a general medical condition, though this is possible. These are much more commonly seen in other psychotic disorders such as schizophrenia.
A 25-year-old man sustained head trauma as a result of his car hitting a tree. After all other causes have been eliminated, it is determined that he has developed psychotic hallucinations because of the head trauma. Which of the following medications is most likely to be helpful in treating this patient’s psychosis?
Risperidone (risperdal)

Psychoses caused by general medical conditions usually respond to antipsychotic medications. There is no indication for the use of drugs used to treat mood disorders (lithium, valproic acid, or sertraline) or benzodiazepines.
A 7-year-old girl has been referred to a psychiatrist by her teacher because she displays inattention, distractibility, and poor concentration and because her poor academic performance resulted in falling grades. Her parents describe a difficulty in following directions, disorganization, and forgetfulness. She does not have any symptoms of depression, psychosis, or developmental problems.

➤ What is the most likely diagnosis?
➤ What are the recommended treatments for this disorder?
➤ Most likely diagnosis: Attention-deficit/hyperactivity disorder (ADHD), inattentive type.

➤ Recommended treatments: Use of a psychostimulant or atomoxetine (strattera) along with behavioral parent training and classroom behavior modification programs.

This patient’s history is fairly typical of attention-deficit disorder, inattentive type (versus the hyperactive type). She shows numerous traits consistent with this disorder, including inattention, making careless mistakes, difficulty paying attention, difficulty following instructions, difficulty organizing tasks, and forgetfulness. This disorder is seen more commonly in girls than in boys. She does not have significant hyperactive symptoms, as do patients with the hyperactive and the combined types; specifically, she does not display behaviors such as squirming, leaving her seat, running or climbing, talking excessively, or constantly being “on the go.”
A 9-year-old boy is referred to a psychiatrist because of poor school performance. He has been tested for learning disabilities but none are present, with an IQ in the high normal range. The teacher reports that it is hard to hold his attention. In addition, he appears hyperactive and fidgety at school, which disrupts the class. However, he does not purposefully go out of his way to disobey the teacher. His parents have noticed no difficulties at home, but his soccer coach has noticed attention problems during practice, and his Sunday school teacher has trouble teaching him because of distractibility. Which of the following is the most likely diagnosis for this patient?
Attention-deficit disorder, combined type.

The diagnostic criteria for ADHD require that the symptoms be present in more than one setting, usually at home and at school. However, this child seems to have evidence of symptoms observed at school, church, and soccer. The child had prominent distractibility and hyperactivity.
A 6-year-old boy with an early, ongoing history of distractibility, hyperactivity, and impulsivity is diagnosed with ADHD. He is treated with methylphenidate. Three weeks later he is brought in and his inattention and hyperactivity is much better. The mother also notes that he has a small bald spot from where he has begun repeatedly rubbing his head. You periodically observe him to suddenly raise his hand to the spot, rub back and forth once, and put his hand down. There is no rash but the area is hairless. The most likely diagnosis is which of the following?
stimulant-induced complex motor tic

Atomoxetine appears to have a lower potential for abuse than Ritalin. Initial use can produce a feeling of sleepiness and grogginess, and the drug does not appear to exhibit its effects for up to 3 weeks. It is not recommended to take atomoxetine on an empty stomach, as side effects of the drug include nausea and vomiting. Atomoxetine is not currently available in a generic form. One of the advantages of this drug is that it affords 24-hour control of ADHD symptoms (ie, it has a longer half-life than Ritalin).
Atomoxetine is a relatively new drug used for the treatment of ADHD. What is the advantage of using atomoxetine over Ritalin?
Atomoxetine appears to have less of a potential for abuse than does Ritalin.

Atomoxetine will not promote the tics. Methylphenidate and amphetamine salts will promote the tics. Buproprion can stimulate dopamine and worsen tics.
A 19-year-old woman has been binging on large quantities of food, above and beyond what most people would eat under similar circumstances. She is embarrassed about the binging and worries that it will make her obese. She then engages in purging behavior, as often as three or four times a week. This behavior accelerates when she is under stress, and the patient feels she has no control over it. The results of her physical examination are normal, and she is of normal weight.

➤ What is the most likely diagnosis for this patient?

➤ What treatment modalities should the psychiatrist recommend?

➤ What areas of the physical and laboratory evaluation should receive special attention?
➤ Most likely diagnosis: Bulimia nervosa.

➤ Best treatment modalities: Nutritional rehabilitation, cognitive-behavioral psychotherapy (CBT), and treatment with an antidepressant (selective serotonin reuptake inhibitor [SSRI]).

➤ Physical examination and laboratory tests: Parotid glands, mouth, teeth for caries, abdominal examination for esophageal or gastric injury, dehydration from laxative use, ipecac-associated hypotension, tachycardia, arrhythmias. Serum electrolytes, magnesium, and amylase levels should also be checked.
Bulimia differs from anorexia nervosa in which way?
Patients with bulimia may not have any symptoms until early adulthood while anorexia typically begins in early adolescence.

Patient with both disorders tend to be high achievers but patients with bulimia tend to be less resistant to getting help, have more alcohol abuse and have more emotional lability than patients with anorexia, who tend to be more emotionally constricted. Bulimia often has a later onset than anorexia.
A 34-year-old woman presents with a 10-year history of episodes in which she eats large quantities of food, such as eight hamburgers and three quarts of ice cream, at a single sitting. Because of her intense feelings of guilt, she then repeatedly induces vomiting. This cycle repeats itself several times a week. She is extremely ashamed of her behavior but says, “I can’t stop doing it.”

On examination, which of the following physical findings is most likely to be seen?
dental caries

The most likely diagnosis for this woman is bulimia nervosa. Physical findings can include dental caries, a round face caused by enlarged parotid glands, or calluses on the fingers resulting from recurrent self-induced vomiting. Lanugo and muscle wasting result from the severe weight loss characteristic of anorexia nervosa.
Which of the following laboratory abnormalities would most likely be found in a patient with bulimia?
hypochloremic-hypokalemic alkalosis

Laboratory abnormalities found in individuals with bulimia nervosa demonstrate hypochloremic-hypokalemic alkalosis resulting from repetitive emesis. Hyperamylasemia and hypomagnesemia are also not uncommonly seen in such patients. Various electrolyte imbalances can occur as a result of frequent laxative abuse. Thyroid abnormalities are not common in individuals with bulimia nervosa.
Which of the following treatment options would be contraindicated among recommended treatment options for a patient diagnosed with bulimia nervosa?

A. Nutritional rehabilitation
B. Cognitive-behavioral psychotherapy
C. Careful use of SSRIs
D. Group psychotherapy
E. Atypical antipsychotic medications to increase her appetite
E - atypical antipsychotics

There is no clinical evidence for the use of atypical antipsychotics in a patient with bulimia.
A 35-year-old man presents to a psychiatrist 1 week after living through a tornado that killed his wife. Since that time he has been “in a daze,” and he describes feelings of numbness and derealization. He sees recurrent intrusive images of the event and tries to avoid thinking about it. He does not sleep well and becomes anxious when he hears a loud noise. His ability to function has been impaired (he is not able to work or call his insurance companies).

➤ What is the most likely diagnosis for this patient?
➤ What should be the next step in his treatment?
➤ Most likely diagnosis: Acute stress disorder.

➤ Next step in treatment: The major initial approach is support, especially facilitating and strengthening family and community support. Prior spiritual or religious affiliation, which can lend meaning to the event and loss, can be very helpful to the individual. Educating the patient and family about expected symptoms and a variety of coping techniques (such as relaxation training) can be very helpful. The use of sedatives or hypnotics in the short term can also be useful.

This patient suffered an acute traumatic episode 1 week prior to his appearance at the psychiatrist’s office. The response to the trauma has lasted more than 2 days but less than 4 weeks. The patient experiences several dissociative symptoms (feeling in a daze, derealization, numbing). He relives the event over and over in his mind (seeing his wife killed and hearing the tor- nado). The patient avoids talking about the trauma to avoid arousing recol- lections of it. He has symptoms of anxiety (insomnia, anxiety when hearing loud noises) that prevent him from functioning well (not working, failing to call his insurance companies to report the losses).
A perception that the environment is somehow different or strange, although the individual cannot account for the changes.
DEREALIZATION
Memory loss of some component of an event, which in the case of acute stress disorder, is usually traumatic.
DISSOCIATIVE AMNESIA
A 28-year-old male assembly-line worker comes in for treatment after developing symptoms a few days after a serious factory accident in which he was knocked unconscious by a machine. He was medically cleared directly following the event but later developed nightmares about the accident. He says he has been very anxious, fears returning to work, and is thinking about seeking disability pay. He has refused to talk to his wife about the incident, because it “makes it worse.” Being around large objects also frightens him, since he is worried he will be hit in the head again and he notes that he thinks about the accident constantly. What is the most likely diagnosis?
Acute Stress Disorder

The most likely diagnosis is ASD, since this patient’s symptoms occurred within 4 weeks of the event and have lasted at least 2 days. If the symptoms had lasted longer than 4 weeks, the diagnosis for a patient with these symptoms would become PTSD. An adjustment disorder is characterized by mood disturbances (anxiety, depression) in response to a difficult situation, but adjustment disorders do not present with the avoidance symptoms (avoiding talking about the incident) and/or reexperiencing of the traumatic event. Generalized anxiety disorder presents with the patient constantly worrying about a wide variety of imagined problems, not connected to an individual traumatic event. Factitious disorder is the feigning of mental or physical illness for primary gain.
Treatment of ASD should focus primarily on which of the following?
mobilizing social supports

Pharmacologic treatments, psychotherapy, and biofeedback are pri- marily interventions for PTSD; the results of research are currently unclear regarding the benefits of debriefing. Mobilizing social supports is the most effective intervention in treating patients with ASD.
Acute stress disorder is diagnosed in a 32-year-old woman who witnessed her fiancee being shot to death in a robbery attempt. She has difficulty sleeping and feels that she is not emotionally attached to anything around her. She also has repetitive flashbacks of the event and avoids going near the location where the incident occurred. Which of the following medications might be helpful to this patient over the short term?
Zolpidem (Ambien)

The use of a hypnotic for insomnia is likely to be helpful to this patient in the short term. Buspirone is used for those patients diagnosed with generalized anxiety disorder, while paroxetine is used in the treatment of major depression. Risperidone is an antipsychotic and valproate is used for mood stabilization of patients with bipolar disorder.
clinical pearls for Acute Stress Disorder
➤ Acute stress disorder occurs within 4 weeks of a traumatic event and lasts at least 2 days for a maximum of 4 weeks.

➤ When symptoms of ASD persist beyond 4 weeks, PTSD must be considered.

➤ Diagnostic criteria for ASD include a response to the traumatic event char- acterized by fear, helplessness, and dissociative symptoms (detachment, decreased awareness of surroundings, derealization, dissociative amnesia).

➤ Individuals with ASD feel detached, feel “unreal,” and can have dissociative
amnesia.

➤ Mobilizing social supports is the first intervention in treating patients with
ASD; most symptoms resolve without pharmacologic treatment.

➤ There are many resilience and risk factors in the development of PTSD or ASD, but most individuals develop symptoms given a traumatic stressor of
sufficient magnitude.
A 54-year-old man with a long-standing history of substance abuse presents to the emergency department with abdominal pain, sweats, diarrhea, and body aches. On initial evaluation, the patient is noted to have a watery nose and eyes, a fever of 100°F (37.8°C), and dilated pupils. His mood is dysphoric. His abdominal examination is benign. Complete blood count, liver function tests, amylase, and lipase are all normal. An upright KUB (kidney, ureter, and bladder) radiograph showed no clear cause for his abdominal pain.

➤ What is the most likely diagnosis for this patient?
➤ What drug can be used to alleviate the patient’s symptoms?
Summary: A 54-year-old man presents with abdominal pain, sweats, diarrhea, and body aches. A physical examination shows pupillary dilation, lacrima- tion, rhinorrhea, and a fever of 100°F (37.8°C). His mood is dysphoric.

➤ Most likely diagnosis: Opioid withdrawal
➤ Medication to help alleviate symptoms: Methadone or clonidine

Shortly after he ceased using his drug of choice after years of heavy use, this patient began to experience classic signs and symptoms of opioid withdrawal. Whereas opiate intoxication causes apathy, psychomotor retardation, constricted pupils, and drowsiness, opiate withdrawal results in nausea and vomiting, muscle aches, lacrimation, rhinorrhea, diarrhea, fever, and dilated pupils. These symptoms can develop within hours or days of the last opiate dose depending on the half-life of the agent and the body’s dependence status. In general, agents with a short half-life tend to induce a rapid, severe withdrawal effect, whereas opiates with a long half-life tend to be associated with a less severe, more gradual withdrawal course. The introduction of clonidine to treat the autonomic hyperactivity in the acute phase, along with the short-term administration of a long-acting opiate such as methadone, is effective.
A 25-year-old man comes to the hospital with symptoms of gastroin- testinal distress, muscle aches, rhinorrhea, lacrimation, and an anxious mood. He states that he “wants to kick this thing once and for all.” Which of the following medications would be most helpful in ameliorating his symptoms?
clonidine (Catapres)

Clonidine can be used to help ease the withdrawal symptoms of opioid withdrawal. It is not an opioid and does not have any addictive properties. Withdrawal from the opioid, however, is not as painless as it otherwise would be if an opioid such as methadone were used. Blood pressure levels should be monitored when clonidine is used. Antabuse or disulfiram is a treatment option for alcoholics. It is not used to treat acute opioid withdrawal. Naloxone is used to counter the effects of life-threatening depression of the central nervous system and respira- tory system from opioid overdose (eg, heroine or methadone overdose). Lorazepam, a benzodiazepine, is commonly used in the treatment of alcohol withdrawal. Haloperidol, an antipsychotic, has no use in treating withdrawal.
A 42-year-old woman is determined to “kick her heroin habit” at home without the use of methadone or any other prescription drug. Of the following over-the-counter medications, which is most likely to be of benefit to this patient as she goes through opioid withdrawal?
Ibuprofen

Ibuprofen can help relieve the muscle cramps that are common to opioid withdrawal.
A 32-year-old man with a long-standing heroin addiction has recently started maintenance treatment with methadone. Three days after starting the methadone regimen, he is experiencing some craving, diarrhea, and mild sweating. His urine toxicology screen is negative for any opiates besides methadone. Which of the following is the most appropriate course of action?
increase the dose of methadone

Clinical signs of withdrawal appearing very early in the maintenance treatment of heroin addiction with methadone are an indication that the dose is not sufficient to ameliorate all the withdrawal symptoms. Because the patient is at great risk of returning to the use of heroin at this point in the process, a dose adjustment upward to prevent craving and withdrawal is appropriate.
Clinical pearls for opioid abuse
➤ A mnemonic that is helpful in remembering the signs and symptoms of opioid withdrawal is SLUD—salivation, lacrimation, urination, and defecation.

➤ Opioid withdrawal is extremely uncomfortable for the patient but gener-
ally is not life-threatening.

➤ Clonidine and methadone (a long-acting opioid) are the two most com-
mon treatments for the relief of opioid withdrawal symptoms.

➤ Loperamide (for loose stools) and promethazine (for nausea and vomiting) are useful adjunctive treatments for the symptoms of opioid withdrawal,
as is ibuprofen for muscle and joint aches.

➤ The presence of dilated pupils and anxiety are often the first signs of opioid withdrawal that can easily be seen in the physician’s office.

➤ Methadone maintenance programs swap one addiction for another, but the social and physical advantages gained make it one of the best choices for treatment of opiate addiction. Other substances of abuse should
always be sought in the presence of heroin addiction.
A 42-year-old woman presents to a primary care physician with a chief complaint of back pain for the past 6 months that began after she was knocked down by a man attempting to elude the police. She states that she has extreme pain on the right side of her lower back, near L4 and L5. The pain does not radiate, and nothing makes it better or worse. She says that since the injury she has been unable to function and spends most of her days lying in bed or sitting up, immobile, in a chair. Immediately after the accident, she was taken to an emergency department where a workup revealed back strain but no fractures. Since then, the patient has repeatedly sought help from a variety of specialists, but the ongoing pain has been neither adequately explained nor relieved. She denies other medical problems, although she mentions a past history of domestic violence that resulted in several visits to the emergency department for treatment of bruises and lacerations.
On mental status examination, the patient is alert and oriented to person, place, and time. She is cooperative and maintains good eye contact. She holds herself absolutely still, sitting rigidly in her chair and grimacing when she has to move even the smallest amount. Her mood is depressed, and her affect is congruent. Her thought processes are log- ical, and her thought content is negative for suicidal or homicidal ideation, delusions, or hallucinations.

➤ What is the most likely diagnosis for this patient?
➤ What is the best approach for this patient?
Summary: A 42-year-old woman has unremitting back pain for 6 months since she was knocked down. The pain is right-sided, located near L4 and L5. There are no exacerbating or alleviating factors, and the pain does not radiate. The patient is nonfunctional since the event. No fractures were found at the time of the accident—a diagnosis of back strain was made. Further workups over the past 6 months show no anatomic or physiologic reason for continued pain. The patient has a history of domestic violence and on multiple occasions was treated in the emergency department for bruises and lacerations. The results of her mental status examination are noncontributory to the diagnosis.

➤ Most likely diagnosis: Pain disorder.

➤ Best approach: Validate the patient’s experience of pain. Explain the role of psychological factors as a cause and consequence of pain. Consider antidepressants and referral to a pain clinic.
A relaxation technique by which patients are trained to induce physiologic changes (most frequently the induction of alpha waves on an electroencephalogram [EEG] or vasodilatation of peripheral capillaries) that result in a relaxation response.
BIOFEEDBACK
A 63-year-old woman returns to her family physician with continuing headaches for 9 months. She describes the pain as “constant . . . always with me,” around her entire scalp. She does not appreciate much variation throughout the day, and she cannot name any aggravating or alleviating factors. Although she occasionally feels light-headed when in severe pain, she denies photophobia, visual changes, nausea, or vomiting. She is especially upset about the headaches as she retired in the past year and has been unable to visit her infant granddaughter. Complete neurologic examinations, computerized tomography, magnetic resonance imaging, laboratory studies, and lumbar punctures have been unremarkable. Which of the following is her most likely diagnosis?
Pain disorder

This patient presents with criteria for pain disorder. She has chronic, unremitting headaches that are the focus of her complaints. They have interfered with her ability to travel, and the onset seems to coincide with her retirement and new grandchild. Her condition is not intentionally produced as in factitious disorder or malingering, nor is there any appreciable secondary gain (avoidance of work, financial compen- sation, etc.). The concern is not on having a serious medical illness as in hypochondriasis or multiple physical complaints as in somatization.
What would be the most useful approach for the patient in the previous question (Pain disorder)?
validation of her experience of pain

One of the most important aspects in management of this disorder is to validate the patient’s experience of pain. An empathic response will serve to strengthen the therapeutic alliance. Conversely, implying that the symptoms are “not real” or denying that “there is anything wrong” will only cause the patient further distress and can actually worsen the pain. Although referral to a mental health professional can be indicated and helpful given the psychological factors present in pain disorder, this subject should first be gently broached with the patient in order to avoid the appearance of not taking the pain seriously. Analgesic medication is generally not helpful and narcotic analgesics should be avoided given their abuse and withdrawal potential.
The patient in the previous question feels that her headaches are now “unbearable.” Which of the following treatments would be the most appropriate?

A. Acetaminophen
B. Biofeedback
C. Lorazepam
D. Nonsteroidal anti-inflammatory
E. Oxycodone
biofeedback

Biofeedback and relaxation techniques have demonstrated efficacy in patients with pain disorder, particularly with headaches. Analgesics are often not helpful in these patients. Potentially addicting medica- tions such as benzodiazepines and opiates should especially be avoided in these individuals given the chronic nature of this illness.
clinical pearls for pain disorder
➤ Patients with pain disorder actually feel pain; it does not help to tell them that “it’s all in your head.”

➤ Pain disorder tends to be a chronic condition; patience, acceptance, and regular visits can promote amelioration of the intensity and frequency of complaints. The patient’s therapeutic relationship with the clinician is very important in the management of this condition.
Summary: A 42-year-old man comes to a psychiatrist with complaints of a depressed mood and difficulty sleeping. His says that his girlfriend recently left him. Although he is obviously upset about the loss of the relationship, he can- not describe her in any specific detail, and they had not been going out together for long. The patient’s speech and manner appear somewhat theatrical and overblown. His affect appears euthymic and full-range, and he appears to be trying to directly engage the (female) interviewer by touching her and asking her direct personal questions. In this manner, he appears to be trying to draw attention to himself by being somewhat seductive. He is shown to have normal thought processes and thought content on a mental status examination.

most likely dx?
best initial tx?
➤ Most likely diagnosis: Histrionic personality disorder.
➤ Best initial treatment: Supportive psychotherapy while he grieves the loss of his girlfriend. Setting a strict limit on his seductive behavior needs to be implemented as well.
A defense mechanism by which an individual deals with emotional conflict or stressors with a breakdown in the usually integrated functions of consciousness, memory, perception of self or the environment, or sensory/motor behavior. For example, a woman who has just been told that her child was killed in an automobile accident suddenly feels as if she is not herself but rather is hearing the events unfold as if they are being told to “someone else.”
DISSOCIATION
A defense mechanism by which individuals deal with emotional conflict or stressors by expelling disturbing wishes, thoughts, or experiences from their conscious awareness. For example, a patient is told that she has breast cancer and clearly hears what she has been told because she can repeat the information back to the physician. However, when she returns home later, she tells her husband that the visit went well but that she cannot remember what she and the physician spoke about during the appointment.
REPRESSION
A 35-year-old woman with histrionic personality disorder has seen her psychotherapist once a week for the past year. During a session, the therapist tells the patient that he is going to be on vacation the following 2 weeks. When he returns from the vacation, the patient tells him that she felt he abandoned her and says, “You didn’t even bother to tell me that you would be away.” This lapse in memory can best be described as which defense mechanism common to patients with histrionic personality disorder?
Repression is a common defense mechanism in patients with histri- onic personality disorder.
A 23-year-old woman with a diagnosis of histrionic personality disorder comes to see her physician for the chief complaint of frequent headaches. As the (male) physician is taking the patient’s history, he notices that she is frequently reaching across the desk to touch his arm as he talks to her, as well as leaning far forward in her seat to be nearer to him. Which of the following responses is the most appropriate from the physician?
Tell the patient that he understands her fear about her headaches, but touching him is not appropriate.

Histrionic patients often display inappropriate or seductive behavior. This is best managed by being tactful and sympathetic to the patient, but firmly and clearly placing boundaries on such behavior.
A 20-year-old woman comes to see a psychiatrist at the insistence of her mother, who states that her daughter just “isn’t herself.” The patient has dressed in brightly colored clothes and worn large amounts of makeup for the past 3 weeks. She acts overtly seductive toward her colleagues at work, is more distractible, and is easily irritated. She also sleeps less, claiming that she “no longer needs it.” Which of the following diagnoses best fits this patient’s presentation?
bipolar disorder, mania

This patient has a new onset of behavior that is unlike her usual personality. It includes dressing in loud clothing and wearing lots of makeup, as well as being seductive. She is distractible, irritable, and needs less sleep than usual. All these symptoms point to a manic episode (assuming neither a medical condition nor a substance can account for the sudden change in functioning).
clinical pearls for histrionic personality disorder
➤ Patients with histrionic personality disorder often appear very dramatic and overemotional. They do not seem to have much depth in either their emotions or their relationships with others. They are uncomfortable when they are not the center of attention.

➤ Patients with histrionic personality disorder use the defense mechanisms of dissociation and repression most commonly.

➤ In interacting with these patients, the physician should use a low key, friendly approach but should watch interpersonal boundaries. He or she should not become caught up in personal or sexual relationships with such patients, who can be quite seductive.

➤ Patients with histrionic personality disorder can be differentiated from those with mania because the latter often develop dramatic, seductive symptoms as new-onset behavior, not as a pervasive pattern. Patients with mania commonly also have vegetative symptoms, such as a decreased need for sleep, and psychotic symptoms as well.
A 9-year-old girl is brought to her pediatrician by her mother because of frequent complaints of headaches and stomach aches for the past 3 to 4 weeks. The mother tells the pediatrician that the girl has also been doing worse in school during this same time period and believes it is a result of the chronic aches. The girl has also missed many days of school because of her complaints. Mother has already taken her to have her vision and hearing checked, neither of which is a problem. On further questioning by the medical student, it is found that the child’s father is enlisted in the army reserve and left for a 6-month assignment in Iraq 5 weeks ago. He e-mails her almost daily, but the mother notes how much the daughter worries about him and whether he is safe or not. When interviewed, the girl also notes that in addition to her worries about her father, she also sometimes cries frequently and feels better when she talks to her friends. She occasionally has a bad dream about her father and feels she sleeps more uneasily as a result.

➤ What is the most likely diagnosis for this patient?
➤ What is the treatment of choice for this disorder?
Summary: A 9-year-old girl presents to her pediatrician with a number of short-term (3-4 weeks) somatic complaints. In addition, she also has some mild symptoms related to mood as well as anxiety as a result of her father’s service commitment. She is able to maintain general functioning, but there does seem to be some decline. She shows evidence of good strengths in that she can express these feelings to others and feels better as a result.

➤ Most likely diagnosis: Adjustment disorder with mixed anxiety and depressed mood

➤ Treatment of choice: Psychotherapy (supportive)
Adjustment disorder is diagnosed in a 45-year-old woman who was fired from a job she held for 20 years. She undergoes supportive psychotherapy. Nine months later, she is seen by her physician, but none of her symptoms have resolved. During this time, she has found another job that is similar to her first position in duties and salary. Which of the following is the most likely diagnosis?
major depressive disorder

The duration requirement for symptoms occurring after the stressor resolved is met for a major depression. Since the stressor has been removed, an ongoing adjustment disorder would be an incorrect answer. There is no evidence for psychosis or manic moods, so options relating to bipolar or schizoaffective disorder would be incorrect. PTSD would not be a viable option because the patient has not had a life-threatening stressor occur.
A 52-year-old man presents to his primary care physician after the death of his wife from breast cancer 2 months ago. He complains of depression, inconsolable sadness, frequent crying, and an inability to focus upon his work and usual activities. Which of the following treatments would likely be most helpful for him?
supportive psychotherapy

Supportive psychotherapy is indicated to help the patient deal with his response to his loss, either on an individual or group setting. Medications are not indicated for bereavement, except perhaps a mild sleep aid if insomnia is a problem. Behavioral modification and/or psy- choanalysis are both unnecessary in this setting, since a much more acute problem which is not behavioral in nature is at issue. The patient does not indicate familial problems (other than his wife’s death) so supportive therapy is the best option.
A 27-year-old woman and her 7-year-old son present to a mental health center for treatment. The patients were passengers in the back of the family car, when they were struck by a semitractor trailer, which killed the father and an older sister. Both mother and son endorse significant depressive symptoms. Which of the following symptoms would most likely differ between the presentations of these two patients?
irritability

In the clinical presentation of children and adolescents, one will often find evidence of irritability or short temper rather than a feeling of sadness or depression. The ability to understand the concept of depression seems to be developmentally mediated.
clinical pearls for adjustment disorder
➤ Adjustment disorder has several different subtypes of symptoms: depressed mood, anxiety, or disturbance of conduct.

➤ Children often feel irritable rather than depressed.

➤ The chronology of the symptoms is very important in making the correct
diagnosis.

➤ The most important treatment modality for adjustment disorder involves
psychotherapy and not a somatic intervention.
A 41-year-old nurse presents to the emergency department with concerns that she has hypoglycemia from an insulinoma. She reports repeated episodes of headache, sweating, tremor, and palpitations. She denies any past medical problems and only takes nonsteroidal anti-inflammatory medications for menstrual cramps. On physical examination, she is a well-dressed woman who is intelligent, polite, and cooperative. Her vital signs are stable except for slight tachycardia. The examination is remarkable for diaphoresis, tachycardia, and numerous scars on her abdomen, as well as needle marks on her arms. When asked about this, she says that she feels confused because of her hypoglycemia.
The patient is subsequently admitted to the medical service. Laboratory evaluations demonstrate a decreased fasting blood sugar level and an increased insulin level, but a decreased level of plasma C-peptide, which indicates exogenous insulin injection. When she is confronted with this information, she quickly becomes angry, claims the hospital staff is incom- petent, and requests that she be discharged against medical advice.

➤ What is the most likely diagnosis?
➤ How should you best approach this patient?
Summary: A 41-year-old female health care worker presents to the emergency department with symptoms typical of an insulinoma, including headache, diaphoresis, palpitations, and tremors. She denies a medical history, although her physical examination demonstrates prior surgeries and injections. When confronted with this evidence, she becomes hostile and asks to leave the hospital.
➤ Most likely diagnosis: Factitious disorder.

➤ Best approach: In order to engage the patient in psychiatric treatment,
attempt to ally with her regarding her compulsion to “be sick.”

Although this patient initially presents with classic symptoms of hypoglycemia possibly caused by an insulinoma, discrepancies are noted in her story, especially her denial of a medical history in light of the numerous scars. Her laboratory evaluations are consistent with the use of insulin, which she undoubtedly injects herself. Specifically, although her insulin levels are increased, her serum C-peptide levels are decreased. When confronted, she becomes hostile and defensive and asks to leave the hospital. No obvious external incentives are present. Thus, it appears that her motivation is merely to be sick as primary gain. The fact that the patient consciously created the hypoglycemia rules out the diagnosis of a somatoform or conversion disorder. The absence of a secondary gain differentiates factitious disorder from malingering. It is useful to note that she is an intelligent woman who works in the health care field, a common scenario for this disorder.
The telling of “tall tales,” or lying, commonly seen in factitious disorder.
PSEUDOLOGIA PHANTASTICA
Factitious disorder, especially involving repeated episodes, seeking admission at different hospitals, and pseudologia phantastica.
MÜNCHHAUSEN SYNDROME
Factitious disorders induced in children by parents, who are usually very cooperative after taking them to the hospital.
MÜNCHHAUSEN SYNDROME BY PROXY
Which of the following is most likely the motivation behind the behavior displayed in factitious disorder?
desire to take on the patient role

The primary desire in factitious disorder is to assume the sick role and be taken care of. In contrast, in malingering, the motivation is to achieve a tangible gain (such as avoiding work, school, or a prison sentence) or to obtain narcotics or financial compensation. If the motivation is unconscious, conversion and other somatoform disorders would need to be considered.
Which type of personality disorder is most likely to occur comorbidly with factitious disorder?
Borderline Personality Disorder

Borderline personality disorder is not uncommon in patients with factitious disorder. Individuals with either of these disorders often have similar histories of abuse, molestation, and emotional neglect. Patients with borderline personality disorder also act out their internal psychological conflicts on an interpersonal level, and they display the chaotic, labile affective state seen in factitious disorder.
Which of the following scenarios is most consistent with factitious disorder?
A. Feigning psychosis to avoid criminal charges
B. Lying about back pain to receive time off from work
C. Pseudoseizures in the context of a family conflict
D. Placing feces in urine to receive treatment for a urinary tract
infection
E. Recurrent fears of having a serious illness
(D) placing feces in urine to receive treatment for a UTI

The hallmark of factitious disorder is intentional feigning of a physical or psychiatric illness in order to assume the sick role. Examples include injecting oneself with insulin to create hypoglycemia, taking anticoagulants to fake a bleeding disorder, and con- taminating urine samples with feces to simulate a urinary tract infection. Lying about back pain in order to avoid work or feigning psychosis to avoid criminal chares is an example of malingering. Pseudoseizures are an example of a conversion disorder. Fear of having a serious disease caused by misinterpretation of bodily sensations is characteristic of hypochondriasis.
Which of the following is the most useful approach for patients with factitious disorder?
establishing a therapeutic alliance

Although there is no specific treatment for factitious disorder, the best way to help these patients is to attempt to establish a therapeutic alliance and a working relationship. Although this can be difficult, only then can the patient’s compulsion to feign illness be addressed and dealt with in a psychotherapeutic environment. Confrontation is necessary in some circumstances, but if an accusatory or judgmental manner is employed, patients flee care and begin the cycle again at another hospital. Prematurely discharging patients from the hospital or referring them to legal services has the same result, although in cases of factitious disorder by proxy (where a caretaker simulates illness in a child), referral to child protective services is necessary because this behavior is considered a form of child abuse. Pharmacotherapy use should be limited unless the patient has a comorbid axis I disorder. Due to abuse potential, medications should be used with caution.
clinical pearls for factitious disorder
➤ Factitious disorder is characterized by the intentional production of physical or psychiatric signs or symptoms in order to assume the sick role.

➤ Factitious disorder is more common in women and in those in the health
care professions.

➤ The course of factitious disorder is chronic, with a pattern of lying, self-
inflicted injuries, repeated hospitalizations, and premature discharges.

➤ The best management of factitious disorder involves early identification, avoidance of unnecessary tests and treatments, empathic understanding of the need to be sick, establishment of a therapeutic working relationship,
and potential referral to a mental health professional.
The patient is a 21/2-year-old boy with new-onset sleep problems who has no significant other history. He wakes at night, screaming with autonomic hyperarousal, and his parents are unable to soothe him. These episodes last a few minutes, after which he goes back to normal sleep. The child has no memory of the events in the morning.

➤ What is the most likely diagnosis for this child?

➤ What treatments would you recommend for this child?
➤ Most likely diagnosis: Sleep terror disorder.

➤ Recommended treatments: Protect the child from injury and do nothing.
The disorder is usually time-limited.

This patient’s presentation is typical for sleep terror disorder, a disorder that is found in 3% of all children and less than 1% of adults and typically manifests itself as emotional and behavioral disturbances at night. These events usually occur early in the nightly sleep cycle during delta (slow-wave) sleep. With sleep terror disorder, the affected child does not remember the episodes in the morning. Fever, sleep deprivation, and central nervous system depressants may increase frequency of sleep terror episodes. Typically, these children have no psychopathology. The episodes are usually self-limiting without treatment, and the prognosis is very good. Reassuring the parents is the usual indicated intervention. Nightmares occur during rapid eye movement (REM)
A young child wakes up in the middle of the night screaming, frightened, and running to his parents room in this state. He is consolable and recalls a frightening dream in the morning at breakfast. In which stage of sleep did this nightmare likely occur?
Rapid eye movement (REM) sleep

Nightmare disorder is a parasomnia usually associated with the REM stage of sleep. Night terrors usually occur during non-REM sleep.
A parent brings his child to the pediatrician’s office because of concerns regarding sleep. The child is 3 years old and often wakes up at night screaming loudly, appearing very frightened, striking out when touched, and inconsolable. She remembers none of this when she wakes the next morning. Which of the following pathologies is likely associated with this condition?
Restless leg syndrome

This case is typical for sleep terrors. Sleep terrors often occur in the presence of another sleep disorder—in particular, restless leg syndrome and sleep-disordered breathing.
The pediatrician orders a sleep study on this patient which documents the presence of sleep-disordered breathing, a commonly concurrent phenomenon with sleep terrors. What treatment for the sleep terror might best be considered at this point?

A. A selective serotonin reuptake inhibitor
B. A benzodiazepine sleeping agent
C. Adenoidectomy or tonsillectomy
D. Reassuring the parents that patient will not harm herself
E. Restraining the patient in bed
Adenoidectomy or tonsillectomy

The best treatment for night terrors with the documented presence of another sleep disorder is definitive treatment of that sleep disorder. In this case, an adenoidectomy or tonsillectomy should be considered in consul- tation with an ear, nose, and throat specialist. Nearly all cases of sleep terrors resolve if the primary sleep disorder is addressed appropriately.
clinical pearls for sleep conditions
➤ Patients with sleep terror disorder need to be protected from injuring themselves during these episodes, but otherwise do not require pharmacologic intervention.

➤ Sleep terror disorder occurs almost exclusively during delta sleep.

➤ Enuresis may be effectively treated with the bell-and-pad method, desmopressin, and imipramine.
A 28-year-old woman has had sleeping problems for the past 2 months that were precipitated by an argument with her boyfriend. Since that time, she is preoccupied with getting enough sleep and is worried and frustrated. She has problems falling and staying asleep. She reports no other medical or psychiatric problems, and the results of her physical examination are normal. She denies the use of drugs or alcohol.

➤ What is the most likely diagnosis for this patient?
➤ What treatment recommendations should be made to this patient?
➤ Most likely diagnosis: Primary insomnia.

➤ Recommended treatment: Sleep hygiene education, stimulus control therapy, relaxation training, and cognitive behavioral therapy should be offered. Helpful medications include ramelteon, trazodone, and benzodiazepine receptor agonists such as zolpidem, zaleplon, and triazolam, although benzodiazepine receptor agonists should in general not be used for more than 2 weeks because tolerance and withdrawal can result.

This patient experienced a psychological upset that interfered with her ability to sleep. Subsequently, she developed a vicious cycle of worrying about whether or not she will be able to sleep, which is invariably followed by a poor night’s sleep. She has no signs or symptoms of a mood or other psychiatric disorder and no evidence of a physical disease or a substance abuse/dependence problem.
A 33-year-old married physician presents to your primary care practice with complaints of “depression.” On interview, he denies pervasive feelings of sadness or anhedonia, and he has not had any change in appetite or weight, or any problems concentrating. He has felt tired much of the time for the past 6 weeks, with ongoing, multiple awakenings during the night. On further questioning, he reveals that these difficulties began when he was involved in a malpractice suit after the death of a patient. He was “up obsessing about it” prior to the trial when his sleep distur- bance began. Although the suit was dropped, he continues to wake up frequently, worrying about not being able to fall back asleep. He denies medical problems, alcohol, or drug use. Which of the following is the most likely diagnosis for this patient?
Primary insomnia

The patient likely suffers from primary insomnia. There is no evidence of sleep apnea or a disconnect between the environment and the circadian rhythm of the patient. Despite the insomnia and fatigue, there is no pervasive depressed mood, anhedonia, or other neurovegetative symptoms suggestive of a major depression.
You offer the previous patient (primary insomnia) a short course of lorazepam, but he declines. Which of the following should you next recommend to help his sleep?

A. Eat a late evening meal.
B. Exercise prior to bedtime.
C. Sleep later on the weekends.
D. Take a hot bath in the evening.
E. Take naps during the day.
Take a hot bath in the evening.

Taking a hot bath near bedtime is an effective technique for inducing sleep in some patients. The other options listed do not help and are actually likely to worsen insomnia.
A patient comes to her physician stating that for the last 6 months, since she started a new job, she has difficulty getting up in time for work. She notes that she is not tired around bedtime, and so she stays up for several hours playing computer games. When she finally does go to sleep, she has time to sleep for only 4 to 5 hours before she has to get up to go to work. She then finds herself groggy in the morning and fatigued throughout the day. This problem is interfering with her work at her job and thus is causing her distress. Prior to starting her new office job, the patient worked evening hours as a bartender and did not have a problem with sleeping. She takes no medications and uses no substances that could explain her sleep problems. The results of her physical examination are normal. Which of the following is the most likely diagnosis for this patient?
Circadian rhythm sleep disorder

This patient is suffering from a delayed-sleep-phase type of circa- dian rhythm sleep disorder. Circadian rhythm sleep disorder is charac- terized by a recurrent pattern of sleep disruption leading to excessive sleepiness and/or insomnia because of the mismatch between the sleep-wake schedule required in a person’s environment (in this case, the demands of the patient’s new job) and her circadian sleep-wake pattern. The sleep disorder must cause distress and must not be caused by a substance, a physical condition, or another mental disorder.
Clinical pearls for primary insomnia
➤ Primary insomnia is characterized by trouble falling asleep and multiple awakenings throughout the night. Individuals with this disorder are often preoccupied with getting enough sleep and become more and more frustrated every night, which further inhibits their ability to sleep.

➤ Primary insomnia is a diagnosis of exclusion because interfering physical and mental disorders must all be ruled out before the diagnosis can be made.

➤ Primary insomnia can be treated to break the cycle of insomnia and worry with ramelteon, trazodone, or short-term use of benzodiazepine receptor agonists.

➤ Stimulus control therapy, relaxation training, and sleep hygiene training can be useful in helping patients with primary insomnia sleep.
A 28-year-old women comes to your clinic after being seen by her primary care doctor and several specialists. She complains that for the last 2 years she has experienced headaches, back and joint pain, abdominal pain with nausea and bloating, numbness and tingling in her upper extremities, and irregular menses. No physical cause for her symptoms can be found. The patient is insistent that she just can’t work anymore due to “her pain” and wants you to sign a sick leave request form. Which of the following might help rule out somatization disorder?

A. Age of this patient.
B. Presence of four pain symptoms.
C. Patient feels she can’t work.
D. Symptoms can’t be explained by a medical condition.
E. Symptoms lasting for 2 years.
Patient feels she can’t work.

Intentionally produced symptoms rule out somatization disorder. When external (secondary) gains such as avoiding work are present, malingering should be considered, and if malingering is indeed the diagnosis, it is likely that the symptoms are being produced intention- ally. The other signs/symptoms in this list are consonant with a somatization disorder, and thus would not help rule it out.
A 35-year-old woman with somatization disorder comes to see a new doctor. Prior to seeing this physician, she obtained complete evalua- tions from at least four clinics. The most important part of the treat- ment plan for this patient is which of the following?
Establishing a schedule for regular visits.

Analgesics are generally not helpful in treating these patients, and they probably cannot be relied on to take psychotropic medications. Patients with somatization disorder have difficulty recognizing affects and so are not candidates for dynamic therapy, especially early in their treatment. The most important treatment intervention is establishing a relationship with one clinician and scheduling brief, regular visits. Increasing the patient’s awareness of possible psychological factors involved in the symptoms is a long-range goal and should be done when and if the patient is willing to see a mental health professional.
For a diagnosis of somatization disorder, which of the following criteria must be met?

A. External motivation for symptoms (avoid work or financial gain).
B. Patient is without significant impairment.
C. Symptoms are intentionally feigned or produced.
D. Symptoms begin prior to age 30.
E. Symptoms last less than 6 months.
(d) symptoms begin prior to age 30

The symptoms must occur prior to age 30 and be present for several years.
clinical pearls for somatization disorder
➤ For somatization disorder the prognosis is generally poor. The clinician should set very modest goals, establishing an ongoing therapeutic relationship and the scheduling of regular appointments as the foundation of treatment.

➤ Even when a diagnosis of somatization disorder is established, the patient can still develop a medical illness over time. The clinician must keep an open mind and at the same time try to avoid unnecessary tests.
A 23-year-old woman experiences auditory hallucinations in which God tells her to kill her baby, and the delusion that the baby is evil and must be killed in order to save the world. This is coupled with a depressed mood and labile affect. Her mood and psychotic symptoms appeared suddenly 24 hours after the delivery of the baby, and she had no apparent prior psychiatric history. The patient does not have a history of drug use/abuse or of medical problems, and there is no known family history.
➤ Most likely diagnosis: Mood disorder not otherwise specified. This patient may be developing a major depression, single episode, postpartum onset, severe with mood-congruent psychotic features.
➤ Next steps: The mother should be allowed to see her child, but only under close observation at all times. Under no circumstances should she be discharged home with the infant. The patient may require hospitalization in a psychiatric inpatient unit if the psychoses and depression do not remit immediately. She should be treated with an antipsychotic agent and possibly an antidepressant or mood stabilizer.
A 29-year-old married female with a prior history of major depression, with postpartum onset, in remission, has given birth to her second child 5 days ago. She describes intense but transient periods of sadness with crying spells since the delivery. While she has been able to enjoy moments with her infant daughter, she worries that she will be “a bad mother.” Her sleep is disrupted with resultant fatigue, but her appetite and concentration are adequate. Although she is concerned that her depression will result in harm to her child, she denies any homicidal ideation. She also denies suicidal ideation or hallucinations. She has no medical problems and is not taking any medications except prenatal vitamins. Which of the following is the most likely diagnosis?
postpartum blues

Although the patient has a history of postpartum depression (a sig- nificant risk factor for future episodes) and is suffering from crying spells, fatigue, and guilt, her likely diagnosis is postpartum blues. Postpartum blues usually peaks within the first week after delivery and resolves by 2 weeks. There is no evidence of a past or current manic episode (consistent with bipolar disorder) or psychotic symptomatology (consistent with postpartum psychosis). While there should be a high suspicion for developing a recurrence of her major depression, this patient does not have many of the associated neurovegetative symptoms, such as anhedonia, appetite change, difficulty concentrating, or suicidal ideation. In addition, major depressive episodes with postpartum onset typically occur 1 to 6 months postpartum.
Which of the following treatment approaches is most appropriate for a patient hospitalized with post-partum blues?
continued observation

While this patient may develop a recurrence of major depression requiring antidepressant therapy, postpartum blues is self-limiting, not necessitating formal treatment. Continued observation and reassurance are all that are recommended. Postpartum psychosis requires treatment with an antipsychotic in combination with an antidepressant or a mood stabilizer, or a course of electroconvulsive therapy.
A 29-year-old woman gives birth to her first child, a boy. One week after the delivery, her husband calls his wife’s obstetrician to tell him that he is concerned about his wife’s behavior. The husband states the wife sits by herself in the bedroom almost continually. She tends to the baby only if reminded to do so by her husband, and when she holds the baby, she does so stiffly. The husband states the wife has told him that the baby is “evil” and that she is “satanic” for having the child. Which of the following should the obstetrician tell the husband?
Come to the emergency room with his wife and child as soon as possible.

Such behavior as reported by the husband sounds disturbingly like a postpartum psychosis. This behavior may result in suicidal and/or homicidal behavior on the part of the mother, which is a psychiatric emergency. The patient should be seen as soon as possible in an emer- gent psychiatric setting (the emergency room).
clinical pearls for post-partum blues vs psychosis
➤ Postpartum (“baby”) blues are a common phenomenon but self-limiting, not requiring treatment.
➤ Treating patients with antidepressants during pregnancy can minimize recurrence of postpartum depression in patients with a history of this disorder.
➤ Postpartum psychosis is a medical emergency, requiring immediate hospitalization of the mother in order to protect the patient and infant.
A 22-year-old Caucasian man with a past diagnosis of schizophrenia, recently released from the hospital on risperidone, has had his dose of this drug increased 2 days prior to the emergency room visit. He presents to the emergency room with the acute onset of a fixed upward gaze bilaterally. The patient remains delusional regarding the cause, namely believing that angels are forcing him to look up to heaven and avoid looking down at hell. His mental status examination is significant for an anxious mood, flat affect, and the above delusions. His physical examination demonstrates a continued, upward gaze of his eyes bilaterally.

➤ What is the most likely diagnosis?
➤ What is the next step in treatment?
➤ Most likely diagnosis: Acute dystonic reaction (an extrapyramidal symptom).

➤ Next step in treatment: Benztropine 2 mg intramuscularly, with a repeat dose in 30 minutes if no improvement.

The patient is a young, Caucasian man with a history of schizophrenia. He has recently been released from the hospital on risperidone, the dose of which was increased 2 days prior to the emergency room visit. He now presents with the acute onset of bilateral upward gaze of his eyes. While his explanation is delu- sional, his history (acute onset, recent medication increase) and physical examination (contraction of superior rectus muscles bilaterally) are consistent with an oculogyric crisis, an acute dystonic reaction caused by the patient’s antipsychotic medication.
A 50-year-old woman with past diagnoses of schizoaffective disorder, bipolar type, complains of “nervous tics.” She denies significant affec- tive symptoms but complains of chronic auditory hallucinations of “whispers” without commands. No suicidal or homicidal ideation is present. On examination, she is noted to be sticking her tongue in and out of her mouth and to have repetitive, rhythmic movements of her hands and feet.
most likely dx?
tardive dyskinesia

This female patient with chronic schizoaffective disorder now demonstrates choreoathetoid movements of her tongue and extremi- ties, consistent with tardive dyskinesia. She has several risk factors for development of tardive dyskinesia, including likely long-term treatment with neuroleptics, female gender, and a mood disorder.
A 27-year-old African American man is admitted for acute psychotic symptoms, consisting of command hallucinations to harm others, paranoid delusions, and agitation. He is begun on olanzapine 30 mg daily. After several days, he becomes calmer and withdrawn. When approached by the nurses, he is found to be lying in bed, eyes open but not responsive. He is noted to be sweating but is resistant to being moved. His vital signs demonstrate temperature of 101.4°F, blood pressure 182/98, pulse 104/min, and respiration 22/min.

most likely dx?
Neuroleptic malignant syndrome

This acutely psychotic patient has been started on antipsychotic medication, namely olanzapine, and he has now developed acute men- tal status changes, diaphoresis, rigidity, and fluctuating vital signs. These are all signs and symptoms consistent with neuroleptic malig- nant syndrome, considered to be a medical emergency. Antipsychotic medications should be discontinued immediately, and supportive measures need to be employed.
A 43-year-old divorced female veteran with schizophrenia is being followed in an outpatient community mental health clinic after being discharged from the hospital. Her medications have subsequently been increased to risperidone 3 mg in the AM and 4 mg in the PM. She has some paranoia and ideas of reference, but she denies auditory or visual hallucinations. Her mental status examination is significant for signifi- cant psychomotor slowing, with little spontaneous speech, but with a coarse tremor of her hands bilaterally. Her gait is wide-based and shuf- fling. Her stated mood is “fine,” although her affect appears blunted, with little expression.

most likely dx?
Parkinsonism

The patient is a middle-aged female veteran with chronic psy- chotic symptoms, just released from the hospital with an increase in her risperidone. She now demonstrates bradykinesia, shuffling gait, masked facies, and a coarse tremor, all consistent with neuroleptic- induced parkinsonism. Risk factors for the development of parkinsonism include female gender and an older age.
A 32-year-old single, Caucasian man is admitted with the diagnosis of psychotic disorder, not otherwise specified, rule-out bipolar disorder. After 10 days, he is finally stabilized on valproic acid 2 g daily and aripiprazole 30 mg daily. The nurses are concerned his medications need to be increased or switched as he has been recently sleeping less and more agitated. Upon examination, he admits to feeling “edgy,” but he denies racing thoughts, increased energy, paranoia, or delusions. He states “I just can’t stop walking; I feel like I’m going crazy.”

most likely dx?
Akathisia

The patient is a young, Caucasian man with psychotic symptoms, rule-out bipolar disorder, stabilized on valproic acid and aripiprazole, but who recently has had worsening insomnia, anxiety, and restless- ness (the “need” to move). Given the improvement in his psychiatric symptoms, his current complaints are likely due to akathisia, a feeling of restlessness or anxiety, usually arising several weeks after treatment with antipsychotic medications. Consideration should be given to either decreasing his antipsychotic dose or adding another medication such as a beta-blocker or benzodiazepine.
Clinical Pearls for side affects of neuroleptics
➤ Extrapyramidal symptoms are neurological symptoms, caused by side effects of antipsychotic medications.

➤ The time course for the development of extrapyramidal symptoms after administration of antipsychotics is as follows:
➤ Dystonic reaction: hours→days
➤ Parkinsonism: days→ weeks
➤ Akathisia: days→ weeks
➤ Tardive dyskinesia: years
➤ Neuroleptic malignant syndrome: anytime


➤ Acute dystonia is treated with anticholinergics or antihistamines.
A defense mechanism in which an individual deals with emotional conflict or stress by attributing exaggerated negative qualities to themselves or to others. This behavior can alternate with idealization. For example, a patient states that her therapist is “the worst doctor in the world.”
DEVALUATION
A defense mechanism in which an individual deals with emotional conflict or stress by attributing exaggerated positive qualities to themselves or to others. This behavior can alternate with devaluation. For example, a patient states that her therapist is “the most empathic person on the planet.”
IDEALIZATION
A defense mechanism in which an individual deals with emotional conflict or stress by refusing to acknowledge some painful aspect of external reality or subjective experience that is apparent to others. For example, a patient hospitalized after a severe heart attack tells a physician he feels “as fit as a fiddle” and jumps out of bed and starts performing jumping jacks. When there is gross impairment in reality testing, the denial can be termed psychotic.
DENIAL
A 22-year-old, single graduate student with narcissistic personality disorder is admitted to a hospital after a car accident in which his right femur is fractured. A medical student has been assigned to follow the patient, but when she enters the room and introduces herself as a medical student, the patient states, “Oh, I wouldn’t let a medical student touch me—I need someone with much more experience than you.” Which of the following statements by the medical student is most likely to lead to a successful interview with this patient?
I’m told that you are a very articulate person, and so I’m hoping you’ll teach me what I need to know.

Appealing to the patient’s narcissism by being admiring most often deescalates the patient as well as improves the therapeutic alliance in these cases.
A 36-year-old man with narcissistic personality disorder calls your office asking for an appointment with the “best therapist in the clinic.” One of his complaints is difficulties in his relationships with his col- leagues. The patient states “They are not giving him the credit he deserves for his accomplishments at the law firm.” What is the most likely reason the patient is seeking treatment?
Anger

Patients with narcissistic personality disorder rarely seek treatment and tend to have little insight into their grandiosity. When these indi- viduals do present for treatment it is usually due to underlying anger or depression resulting from being belittled or not receiving the admiration to which they feel entitled.
The patient in the previous question has now been seeing a therapist twice weekly for the last year. The therapist and the patient have a good working alliance. During one therapy session, the therapist comes to the session 4 minutes late. He apologizes to the patient, stating that he had an emergency involving another patient. During the session, the patient notes that the therapist “isn’t as sharp as some of the therapists I hear on the talk shows.” Which of the following defense mechanisms is the patient using?
devaluation

The patient defends against his feelings of hurt and anger toward the therapist by using devaluation. Devaluation, along with idealiza- tion and denial are considered primitive (lower-functioning) defense mechanisms used by patients with personality disorders such as narcissistic and borderline.
clinical pearls for narcissistic personality disorder
➤ Patients with narcissistic personality disorder show a pervasive sense of grandiosity and entitlement in their thoughts and behavior. They are very seldom capable of true empathy with others and often manipulate them for personal gain.
➤ Clinicians should try to maintain an admiring stance with these patients. Tact is important as well, as these individuals handle criticism poorly.
➤ Defense mechanisms in patients with narcissistic personality disorder include denial, devaluation, and idealization.